Тестовые задания по клинической фармакологии для ординаторов. Тест по Клинической фармакологии система подготовки к тестам Gee Test oldkyx com Список вопросов по Клинической фармакологии

1.Как называется раздел фармакологии, изучающий всасывание, распределение, биотрансформацию и выведение лекарственных средств?

Фармакокинетика.

Фармакодинамика.

2.Как называется раздел фармакологии, изучающий виды действия лекарственных средств, фармакологические эффекты, механизм действия?

Фармакодинамика.

Фармакокинетика.

3.Основной механизм всасывания лекарственных средств в желудочно-кишечном тракте:

Активный транспорт.

Облегчённая диффузия.

Пассивная диффузия через мембраны клеток.

Пиноцитоз.

4.Основное место всасывания лекарств – слабых оснований:

Тонкий кишечник.

5.Основное место всасывания лекарств – слабых кислот:

Тонкий кишечник.

6.Какой способ введения лекарственных средств обеспечивает 100 %-ю биодоступность?

Внутримышечный.

Ректальный.

Внутривенный.

Через рот.

7.Как изменится всасывание лекарственных средств – слабых кислот при уменьшении кислотности желудочного сока?

Увеличится.

Уменьшится.

8.Как изменится всасывание лекарственных средств – слабых оснований при уменьшении кислотности желудочного сока?

Увеличится.

Уменьшится.

9.Путём пассивной диффузии через биологические мембраны легко транспортируются вещества:

Липофильные.

Полярные.

Гидрофильные.

10.Энтеральный путь введения лекарственных средств:

Внутримышечный.

Ингаляционный.

Сублингвальный.

Внутривенный.

11.Парентеральный путь введения лекарственных средств:

Через рот.

В прямую кишку.

Подкожный.

Сублингвальный.

12.Где в основном происходят, всасывание большей части лекарственных средств?

В ротовой полости.

В желудке.

В тонком кишечнике.

В толстом кишечнике.

13.Внутривенно можно вводить:

Масляные растворы.

Нерастворимые соединения.

Осмотически активные соединения.

Микрокристаллические взвеси.

Нерастворимые соединения.

14.Какое функциональное изменение в организме вызывают сердечные гликозиды при сердечной недостаточности?

Возбуждение.

Угнетение.

Тонизирование.

Успокоение.

15.Какое функциональное изменение в организме вызывает средство, понижающее артериальное давление при артериальной гипертензии?

Возбуждение.

Угнетение.

Тонизирование.

Успокоение.

16.Как называется накопление в организме лекарственного средства при его повторных введениях?

Функциональная кумуляция.

Сенсибилизация.

Материальная кумуляция.

Тахифилаксия.

17.Толерантность – это:

Аллергическая реакция организма на повторное введение лекарства.

Уменьшение фармакологического эффекта на повторное введение лекарства.

Непреодолимое стремление к повторному приёму лекарства.

18.Снижение эффекта при введении лекарственных средств через короткие промежутки времени – это:

Тахифилаксия.

Идиосинкразия.

Сенсибилизация.

Пристрастие.

19.Побочный эффект, который может возникнуть только при повторных введениях лекарственных средств:

Идиосинкразия.

Тератогенное действие.

Мутагенное действие.

Привыкание.

20.Побочный эффект, который может возникнуть только при применении психотропных средств:

Идиосинкразия.

Пристрастие.

Привыкание.

Сенсибилизация.

21.Определите вид взаимодействия лекарственных средств: больному при отравлении мускарином проведено промывание желудка взвесью активированного угля:

Суммированный синергизм.

Химический антагонизм.

Конкурентный антагонизм.

Физический антагонизм.

22.Мутагенное действие – это:

23.Тератогенное действие – это:

Повреждение генетического аппарата зародышевой клетки.

Нарушение дифференцировки тканей плода, вызывающее различные аномалии.

Побочный эффект, возникающий в первые 12 недель после оплодотворения и вызывающий гибель зародыша.

24.Эмбриотоксическое действие – это:

Повреждение генетического аппарата зародышевой клетки.

Нарушение дифференцировки тканей плода, вызывающее различные аномалии.

Побочный эффект, возникающий в первые 12 недель после оплодотворения и вызывающий гибель зародыша.

МОДУЛЬ 3 КЛИНИЧЕСКАЯ ФАРМАКОДИНАМИКА. ПАЦИЕНТ И ЛЕКАРСТВО

МОДУЛЬ 3 КЛИНИЧЕСКАЯ ФАРМАКОДИНАМИКА. ПАЦИЕНТ И ЛЕКАРСТВО

После освоения темы студент должен знать

1. Определение фармакодинамики.

2. Определения антагонистов, агонистов, частичных агонистов.

3. Типы молекул-мишеней лекарственных средств (рецепторы, ферменты, ионные каналы).

4. Виды фармакологического ответа: ожидаемый фармакологический ответ, гиперреактивность, тахифилаксия, идиосинкразия.

5. Принципы разработки программ контроля эффективности лекарственных средств.

6. Острый фармакологический тест (понятие, назначения, показания, правила проведения).

7. Методы оценки влияния лекарственных средств на качество жизни.

8. Методологию сбора фармакологического анамнеза (понятие, клиническое значение, правила сбора, интерпретация).

9. Приверженность больного лечению - комплаентность (понятие, факторы, влияющие на приверженность лечению, методы повышения приверженности больного лечению).

10. Ответственное самолечение.

После освоения темы студент должен уметь

1. Интерпретировать информацию о фармакодинамике лекарственных средств (из инструкции и ТКФС) для индивидуализированного выбора лекарственных средств.

2. Разрабатывать программу контроля эффективности лекарственных средств с учетом их фармакологических эффектов.

3. Оценивать влияние лекарственных средств на качество жизни.

4. Проводить острый фармакологический тест, интерпретировать его результаты для выбора лекарственных средств.

5. Собирать и интерпретировать фармакологический анамнез.

6. Проводить мероприятия, повышающие приверженность больного к медикаментозному лечению.

Литература, необходимая для освоения темы

Основная

Кукес В.Г. Клиническая фармакология. - М.: ГЭОТАР-Медиа, 2008. - С. 80-94, 95-117.

Дополнительная

Новик А.А., Ионова Т.И. Руководство по исследованию качества жизни в медицине. - М.: Олма медиа групп, 2007. - 320 с.

Сергеев П.В., Шимановский Н.Л., Петров В.И. Рецепторы. - Волгоград, 1999. - 640 с.

Лекция, которую необходимо прослушать для освоения темы

Кукес В.Г. «Введение в клиническую фармакологию. Фармакодинамика».

http://lech.mma.ru/clinpharm/ucheb/pharm/lekt/1

Выполните тестовые задания для самоконтроля

Выберите один или несколько правильных ответов.

I. Фармакодинамика включает:

A. Процессы всасывания, распределения, метаболизма и выведения ЛС.

Б. Механизм действия, локализацию действия и виды действия ЛС.

B. Движение ЛС в организме больного. Г. Физико-химические свойства ЛС.

II. Установите соответствие терминов и их определений:

A. Агонисты.

Б. Антагонисты.

B. Частичные агонисты.

1. ЛС, связывающиеся с теми же рецепторами, что и эндогенные медиаторы, вызывающие «нулевой эффект».

2. ЛС, связывающиеся с теми же рецепторами, что и эндогенные медиаторы, вызывающие эффект равный или больший эффекту данного медиатора.

3. ЛС, связывающиеся с теми же рецепторами, что и эндогенные медиаторы, вызывающие эффект больше нуля, но меньше эффекта данного медиатора. III. Установите соответствие терминов и их определений:

A. Идиосинкразия. Б. Тахифилаксия.

B. Толерантность.

1. Снижение терапевтического эффекта, наблюдающееся при повторном применении ЛС.

2. Генетически обусловленная извращенная реакция на определенный лекарственный препарат, проявляющаяся повышенной чувствительностью к нему и/или длительным эффектом и связанная с генетически детерминированным дефектом ферментных систем.

3. Снижение терапевтического эффекта, наблюдающееся при длительном (многократном) применении ЛС.

Задание 3.1. Приведите примеры лекарственных средств - агонистов и антагонистов различных типов (а точнее классов) рецепторов, заполнив схему на рис. 3.1. В центре в прямоугольниках укажите рецепторы, сверху и снизу в стрелках - примеры лекарственных средств, являющихся их агонистами или антагонистами.

Более подробная информация о взаимодействии ЛС с молекулами-мишенями в лекции R. Leurs «Drug-receptor interaction» (на английском языке): www.ux1.eiu.edu/~cfthb/classes/4790/pdfs/ Drug-Receptor_Interactions.pdf

Зачем врачу нужна информация о фармакодинамике лекарственных средств и как ее интерпретировать

Фармакодинамика - это раздел клинической фармакологии, изучающий механизм действия, локализацию действия и виды действия ЛС у больного. В упрощенном виде фармакодинамика позволяет понять, что ЛС делает с организмом больного. При этом выделяют основной или терапевтический эффект ЛС - фармакологический

Рис. 3.1. Лекарственные средства - агонисты и антагонисты различных рецепторов

эффект, который направлен на достижение поставленных целей лечения. Все фармакологические эффекты, не относящиеся к основному, расцениваются как побочные, которые могут быть не только «отрицательными» для здоровья больного (нежелательные побочные реакции), но и «положительными». Информация о фармакодинамике ЛС содержится в разделе «Фармакологическое действие» инструкции и ТКФС и предназначена для врача. На основании данной информации у врача формируется представление о значении фармакодинамических эффектов в развитии терапевтического и нежелательных лекарственных реакций (табл. 3.1), что определяет показания и противопоказания к применению ЛС, а следовательно, и выбор данного ЛС. Таким образом, врач должен понимать клиническое значение изложенной информации для индивидуализированного выбора как группы ЛС, так и отдельных ЛС (в том числе с учетом результатов острого фармакологического теста), разработки программы контроля эффективности лечения (в том числе оценки динамики качества жизни больного на фоне применения ЛС).

Таблица 3.1. Значение информации о фармакодинамике бисопролола для понимания его терапевтических

и неблагоприятных побочных реакций

Лекарственное средство

Молекула-мишень и характер взаимодействия с ней

(агонист или антагонист)

Локализация действия

Виды действия

Фармакодинамический эффект

Терапевтический эффект и соответствующее показание к применению

Неблагоприятная побочная реакция

Бисопролол

β1-адренорецеп- торы

Кардиомиоциты

Отрицательный ино-, дромо-, хроно-, батмотропный. Снижение потребности миокарда в кислороде.

Гипотензивный

Антиангинальный - ИБС. Снижение ЧСС, антиаритмический эффект - тахиаритмии; ХСН.

Гипотензивный -

артериальная

гипертензия

Брадикардия. Угнетение АВ-проводимости

В больших

β 2 -адренорецеп-

Поджелудочная железа, скелетные мышцы, гладкая мускулатура периферических артерий, бронхов и матки

Снижение тонуса бронхов; гипергликемический эффект; снижение периферического кровообращения

Не используется

Бронхоспазм Нарушение углеводного обмена, периферического кровотока

Выполните задание для самостоятельной работы

Задание 3.2. Опишите фармакодинамику предложенного преподавателем ЛС или ЛС по своей будущей специальности, заполнив таблицу по аналогии с табл. 3.1. Используйте раздел «Фармакологическое действие» ТКФС.

Как знание особенностей фармакодинамики помогает врачу выбрать конкретное лекарственное средство из группы?

Знание особенностей фармакодинамики ЛС лежит в основе выбора конкретного ЛС внутри выбранной группы в соответствии с принципом совпадения профиля ЛС (особенности фармакодинамики ЛС из раздела «Фармакологическое действие» инструкции и ТКФС) и профиля пациента (индивидуальные особенности пациента). В качестве примера представлен выбор ЛС из группы β-адреноблокаторов для лечения артериальной гипертензии на основе особенностей фармакодинамики (табл. 3.2).

Таблица 3.2. Выбор лекарственного средства из группы β-адреноблокаторов для лечения артериальной гипертензии на основе особенностей фармакодинамики

Особенность фармакодинамики

Принципы выбора

Пропранолол Карведилол

Блокада β Γ

и β2-адренорецепторов

Выбираем при сопутствующей портальной гипертензии. Не выбираем при сопутствующей ХОБЛ, сахарном диабете, гиперлипидемии, у больных молодого и среднего возраста (сексуально-активных)

Атенолол Метопролол Бисопролол Небиволол

Селективная блокада βl-адренорецепторов

Выбираем при сопутствующей ХОБЛ (особенно бисопролол и небиволол), у больных молодого и среднего возраста (сексуальноактивных) (особенно бисопролол и небиволол), сахарном диабете в стадии компенсации (при хорошо подобранной сахароснижающей терапии)

Окончание табл. 3.2

Особенность фармакодинамики

Принципы выбора

Карведилол Небиволол

Наличие дополнительных вазодилатирующих свойств: у небиволола за счет способности стимулировать высвобождение оксида азота в эндотелии сосудов, а у карведилола за счет α- адрено блокирующего эффекта

Выбираем при сопутствующем облитерирующем атеросклерозе сосудов нижних конечностей (за исключением случаев критической ишемии)

Карведилол

Метопролол

(в виде сук-

Бисопролол

Небиволол

Доказанное в рандомизированных исследованиях положительное влияние на прогноз у больных с хронической сердечной недостаточностью за счет снижения активности симпатикоадреналовой системы

Выбираем при сопутствующей хронической сердечной недостаточности в стадии компенсации (при отсутствии признаков задержки жидкости). При этом небиволол выбираем только больным с хронической сердечной недостаточностью старше 75 лет (есть доказательства эффективности этого ЛС только для этой категории пациентов). Следует отдавать предпочтение карведилолу при сочетании хронической сердечной недостаточности и артериальной гипертензии III степени (карведилол обладает более выраженным антигипертензивным эффектом за счет α-адреноблокирующего эффекта)

Выполните задание для самостоятельной работы

Задание 3.3. Используя раздел «Фармакологическое действие» ТКФС, разработайте принципы выбора ЛС внутри группы ЛС по теме практического занятия (группа ЛС предлагается преподавателем)

или из вашей будущей специальности. Для этого заполните таблицу по аналогии с табл. 3.2.

Выбор ЛС из группы должен осуществляться не только на основе особенностей фармакодинамики, но и особенностей фармакокинетики, учет которых также способствует выбору индивидуализированного режима дозирования (см. модуль 2).

Как разрабатывать индивидуальную программу контроля эффективности лекарственного средства?

В процессе применения ЛП врач должен осуществлять контроль эффективности, т.е. за степенью достижения поставленных целей лечения (первичных и вторичных). Для этого необходимо разработать программу контроля эффективности, которая будет представлять собой методы (клинические, лабораторные и инструментальные), оценивающие развитие терапевтических эффектов ЛП, а также сроки их выполнения и уровень изменения результатов этих методов на фоне лечения (табл. 3.3). Выбранные методы в программе контроля эффективности ЛП должны давать информацию врачу о степени достижения первичных и вторичных целей лечения у больного в зависимости от показаний к применению ЛП. На основе получаемых результатов врач принимает соответствующие решения: корректирует дозы ЛП, продолжает применение ЛП, отменяет ЛП, присоединяет другие ЛП. Эти решения врач принимает также в соответствии с результатами программы контроля за безопасностью (см. модуль 4). Программы контроля эффективности и безопасности реализуются параллельно и взаимно дополняют друг друга. Для разработки программы контроля эффективности ЛП следует использовать информацию из разделов «Фармакологическое действие» и «Показания» инструкции и ТКФС.

Выполните задание для самостоятельной работы

Задание 3.4. Разработайте программу контроля эффективности применения ЛС, предложенного преподавателем, или самостоятельно выбрав ЛС по своей будущей специальности. Заполните таблицу по аналогии с табл. 3.3. Используйте раздел «Фармакологическое действие» ТКФС.

Таблица 3.3. Методы контроля эффективности лечения больного с обострением язвенной болезни двенадцатиперстной кишки

Показание у данного пациента

Лекарственное средство

Методы оценки эффективности

Заживление язвенного дефекта

Лекарствен- ное средство 1 - рабепразол

Механизм 1 . Инактивирует сульфгидрильные группы Н+/К+-АТФазы. Блокирует заключительную стадию секреции HCl, снижая содержание базальной и стимулированной секреции. Механизм 2. Проникает в париетальные клетки желудка и концентрируется в них, оказывая цитопротекторное действие и увеличивая секрецию гидрокарбоната

Клинические методы: уменьшение болевого синдрома или его купирование, устранение изжоги, отрыжки. Лабораторные методы: лабораторные маркеры эрадикации Н. pylory (при совместном применении с антибиотиками) - см. ниже. Инструментальные методы: ЭГДС - рубцевание язвы, уменьшение ее размера. рН-метрия - повышение внутрижелудочного рН

Эрадикация Н. pylori

Лекарственное средство 1 - рабепразол

Механизм 1. Увеличивает концентрацию антибактериальных средств в слизистой оболочке желудка.

Механизм 2. Повышает рН, увеличивая активность антибиотиков (амоксициллина, кларитромицина). Механизм 3. Антихеликобактерный эффект (подавляют рост Н. pylori in vivo, действуя на систему АТФазы бактерий)

Клинические методы: отсутствуют. Лабораторные методы: уреазный дыхательный тест через 3-4 нед после окончания терапии.

Инструментальные методы:

гистологическое исследование биоптата;

уреазная проба с биоптатом

Окончание табл.3.3

Показание у данного пациента

Лекарственное средство

Механизм действия лекарственного средства, соответствующий показанию

Методы оценки эффективности

Кларитромицин

Механизм 1. Подавляет синтез белка путем обратимого связывания с 50S- субъединицей рибосом Н. pylori, оказывая бактериостатический эффект

Амоксициллин

Механизм 1. Нарушают образование клеточной стенки бактерий за счет необратимого связывания с пенициллиносвязываю- щими белками, оказывая бактерицидный эффект

Как использовать оценку качества жизни в качестве клинического метода контроля эффективности лекарственного препарата?

Важным клиническим методом контроля эффективности ЛП считается оценка качества жизни в динамике. Качество жизни - это интегральный показатель, включающий в себя несколько компонентов:

Функциональное состояние (работоспособность, толерантность к физической нагрузке, выполнение домашней работы);

Симптомы, связанные с заболеванием или его лечением (боль, одышка, побочные эффекты ЛС);

Психическое состояние (депрессия или возбуждение, которые могут быть как следствием самого заболевания, так и применения ЛС);

Социальная активность;

Половая функция;

Удовлетворение медицинской помощью.

Факторы, ухудшающие качество жизни, можно разделить на две группы:

Факторы, связанные с заболеванием (симптомы заболевания, мешающие жить);

Факторы, связанные с медикаментозной терапией (неудобство применения ЛП, развитие нежелательных лекарственных реакций).

Степень выраженности факторов, связанных с заболеваниями, оценивают с помощью опросников. При этом существуют универсальные опросники для оценки качества жизни у больных с любыми заболеваниями, однако они, как правило, очень громоздкие, их заполнение и интерпретация занимают много времени у пациентов и врачей, поэтому разработаны специальные опросники для больных наиболее распространенными заболеваниями. Больной должен самостоятельно ответить на вопросы опросника, а врач подсчитывает общее количество баллов в соответствии с ответами пациента. Динамика суммы баллов на фоне лечения ЛП и будет методом контроля эффективности ЛП.

Универсальный опросник качества жизни EQ-5D (на русском языке): http://forum.disser.ru

Более подробная информация об оценке качества жизни находится в статье: Мясоедова Н.А., Тхостова Э.Б., Белоусов Ю.Б. Об оценке качества жизни при различных сердечно-сосудистых заболеваниях: http://www.trimm.ru/php/content.php?group=2&id=3726

Анализ факторов, связанных с медикаментозной терапией, позволяет провести коррекцию применения ЛП в процессе лечения.

Например, проанализируем качество жизни пациента 53 лет с диагнозом: ИБС; постинфарктный кардиосклероз. Артериальная гипертензия 2-й степени, III стадии, очень высокого риска. ХСН ΙΙ ФК по NYHA. Пациент получает эналаприл 5 мг 2 раза в сутки, бисопролол 2,5 мг 1 раз в сутки, фуросемид 40 мг 2 раза в неделю, спиронолактон 25 мг 1 раз в сутки. Пациент жалуется на одышку при ходьбе 300-500 м, подъеме на 4-й этаж (живет на 5-м этаже в доме без лифта), примерно 1 раз в неделю возникает ночная одышка; невозможность заниматься физическим трудом и спортом

(ранее работал автомехаником, увлекался лыжным спортом); в день приема фуросемида вынужден оставаться дома из-за диуретического эффекта, поэтому вынужден перейти на менее оплачиваемую работу со сменным графиком; также отмечает небольшой сухой кашель (от назначения группы блокаторов ангиотензиновых рецепторов отказался ввиду их более высокой стоимости). Факторы, ухудшающие качество жизни больного, представлены в табл. 3.4.

Таблица 3.4. Факторы, ухудшающие качество жизни больного хронической

сердечной недостаточностью

Оценка факторов, ухудшающих качество жизни, связанных с заболеваниями у данного пациента, проводится с помощью специального опросника для больных хронической сердечной недостаточностью, так называемый Минессотский опросник (табл. 3.5). Снижение суммы баллов по этому опроснику на фоне лечения перечисленными ЛП будет свидетельствовать об эффективности терапии. Влияние на качество жизни выявленных факторов, связанных с медикаментозным лечением, в данном случае можно уменьшить путем коррекции терапии:

Заменить прием фуросемида 2 раза в неделю на ежедневный прием гидрохлоротиазида в дозе 25 мг в сутки;

Заменить инигибитор АПФ эналаприл на антагонист ангиотензиновых рецепторов лозартан, который гораздо реже вызывает сухой кашель, но аналогичен по эффективности;

Назначить комбинированный препарат, содержащий в одной таблетке лозартан и гидрохлоротиазид в соответствующих дозах.

Таблица 3.5. Минессотский опросник оценки качества жизни больных с хронической сердечной недостаточностью

Мешала ли вам сердечная недостаточность жить так, как хотелось бы в течение последнего месяца из-за:

1. Отеков голеней, стоп

0, 1, 2, 3, 4, 5

2. Необходимости отдыхать днем

0, 1, 2, 3, 4, 5

3. Трудности подъема по лестнице

0, 1, 2, 3, 4, 5

4. Трудности работать по дому

0, 1, 2, 3, 4, 5

5. Трудности с поездками вне дома

0, 1, 2, 3, 4, 5

6. Нарушений ночного сна

0, 1, 2, 3, 4, 5

7. Трудности общения с друзьями

0, 1, 2, 3, 4, 5

8. Снижения заработка

0, 1, 2, 3, 4, 5

9. Невозможности заниматься спортом, хобби

0, 1, 2, 3, 4, 5

10. Сексуальных нарушений

0, 1, 2, 3, 4, 5

11. Ограничений в диете

0, 1, 2, 3, 4, 5

12. Чувства нехватки воздуха

0, 1, 2, 3, 4, 5

13. Необходимости лежать в больнице

0, 1, 2, 3, 4, 5

14. Чувства слабости, вялости

0, 1, 2, 3, 4, 5

15. Необходимости платить

0, 1, 2, 3, 4, 5

16. Побочного действия лекарств

0, 1, 2, 3, 4, 5

17. Чувства обузы для родных

0, 1, 2, 3, 4, 5

18. Чувства потери контроля

0, 1, 2, 3, 4, 5

19. Чувства беспокойства

0, 1, 2, 3, 4, 5

20. Ухудшения внимания, памяти

0, 1, 2, 3, 4, 5

21. Чувства депрессии

0, 1, 2, 3, 4, 5

Варианты ответов: 0 - нет; 1 - очень мало... 5 - очень много (наивысшее качество жизни - 0 баллов; наиболее низкое - 105 баллов).

Выполните задание для самостоятельной работы

Задание 3.5. Перечислите факторы, определяющие качество жизни у курируемого больного, заполнив таблицу, аналогичную табл. 3.5.

Выполните задание для самостоятельной работы

Задание 3.6. Найдите или разработайте самостоятельно опросник оценки качества жизни больных с заболеванием по вашей будущей специальности. Используйте в качестве примера опросник для оценки качества жизни больных хронической сердечной недостаточностью (Минессотский опросник; см. табл. 3.5).

Что такое острый фармакологический тест и как интерпретировать его результаты для индивидуализированного выбора лекарственных средств?

Острый фармакологический тест - это анализ однократного применения ЛС у курируемого больного, позволяющий прогнозировать терапевтический эффект и развитие нежелательных лекарственных реакций при применении ЛП.

В клинической практике широко применяются диагностические фармакологические тесты: проба с добутамином для выявления ишемии миокарда, проба с АКТГ и дексаметазоном при диагностике надпочечниковой недостаточности, проба с неостигмином при диагностике миастении, диазепамовая проба в диагностике аффективных психозов, проба с бронхолитиком при проведении спирометрии. В ряде случаев фармакологический тест помогает не только подтвердить диагноз, но и определить потенциальную эффективность лечения, например, очевидно, что при положительном тесте с сальбутамолом бронхолитики из группы β2~адреномиметиков будут эффективны.

Понятие острый фармакологический тест подразумевает оценку эффективности и безопасности ЛС при известном диагнозе. При этом используются выбранные методы контроля эффективности и безопасности. Для проведения острого фармакологического теста врачу нужна информация о фармакодинамике ЛС из раздела «Фармакологическое действие» и о фармакокинетике ЛС из соответствующего раздела инструкции по медицинскому применению

ЛС или ТКФС.

Правила проведения острого фармакологического теста разберем на примере. Врач-терапевт вызван на консультацию в неврологическое отделение к пациенту 70 лет с диагнозом: атеросклероз сосудов головного мозга. Атеросклеротический стеноз внутренних сонных артерий. Дисциркуляторная энцефалопатия II стадии в связи с повышением артериального давления до 175/100 мм рт.ст., сопрово-

ждающегося умеренной головной болью. Из анамнеза известно, что у пациента дважды за последние 7 лет наблюдалось повышение АД до 160/100 мм рт.ст., привычное АД составляет 140/80, гипотензивная терапия ранее не проводилась. Исходная ЧСС - 86 в мин. Каким образом необходимо проводить подбор гипотензивной терапии с помощью острого фармакологического теста? В данном случае пациент имеет высокий риск развития нарушения мозгового кровообращения при избыточном снижении АД, следовательно, со стороны врача требуется усиленный контроль безопасности проводимой терапии, уместно проведение острого фармакологического теста.

Пациенту был назначен бисопролол в дозе 2,5 мг.

Проведен контроль АД и ЧСС:

Через 1 ч - 170/95 мм рт. ст; 70 в мин;

Через 2 ч - 160/90 мм рт.ст.; 68 в мин;

Через 3 ч - 140/90 мм рт.ст.; 66 в мин;

Через 4 ч - 110/70 мм рт.ст.; 55 в мин;

Через 6 ч - 115/70 мм рт.ст.; 57 в мин;

Через 12 ч - 160/70 мм рт.ст.; 58 в мин.

Согласно ТКФС, максимальная концентрация бисопролола достигается через 2-4 ч после приема. В данном случае через 4 и 6 ч у пациента наблюдалось избыточное снижение АД, а через 12 ч терапия была недостаточной. Следовательно, начальную дозу бисопролола следует уменьшить, а через 8-10 ч выполнить дополнительный прием препарата.

Что такое фармакологический анамнез и как осуществлять его сбор?

Сбор фармакологического анамнеза - обязательная процедура для врача. Фармакологический анамнез - совокупность сведений о ранее принимавшихся больным лекарственных средствах, способах их применения, дозах, эффективности, нежелательных побочных реакциях, признаках непереносимости, лекарственной зависимости. Фармакологический анамнез позволяет выявить эффективные ранее ЛС и/или ЛС, вызывавшие развитие нежелательных лекарственных реакций. Это может определять выбор как групп, так и конкретных ЛП и их доз. Для этого врачу необходимо:

Выявить, какие ЛС пациент не переносит;

Определить, как конкретно проявлялась непереносимость лекарственных средств;

При этом необходимо обратить внимание, что пациенты часто по-своему трактуют вопросы врача, например под аллергией на нитроглицерин многие пациенты понимают головную боль;

Установить причину непереносимости:

Истинную аллергическую реакцию и степень ее тяжести. Например, при нетяжелой аллергической реакции на пенициллины (крапивница) возможно назначение β-лактамных антибиотиков из группы цефалоспоринов, а при отеке Квинке или анафилактическом шоке все препараты из этого класса противопоказаны;

Развитие тяжелых нежелательных реакций, при этом необходимо выяснить правильность режима дозирования и приема препарата, учесть возможные взаимодействия. Пример - гликозидная интоксикация при приеме дигоксина в суточной дозе 0,25 мг совместно с фуросемидом;

Развитие нетяжелых, но нарушающих качество жизни пациента НПР, например кашель при приеме ингибиторов АПФ;

Неправильный режим дозирования и/или приема ЛС. Например, слишком высокая доза гипотензивного ЛС может привести к ортостатическому коллапсу; разжевывание ретардированных форм нифедипина может сопровождаться выраженной тахикардией, слабостью, приливами;

Оценить эффективность проводимой ранее терапии и выявить возможные причины неэффективности:

Нерегулярный прием препарата (следует обратить внимание на то, что некоторые пациенты, говоря о том, что принимают лекарство постоянно, имеют в виду тогда, когда чувствуют себя плохо);

Недостаточная доза/кратность приема препарата. Например, прием амоксициллина/клавулановой кислоты в дозе 625/125 мг 2 раза в сутки не обеспечивает необходимого антибактериального эффекта;

Неправильная техника введения ЛС. Например, ингаляционные бронхолитические препараты могут быть неэффективны при неправильном выполнении ингаляции препарата, соответственно, вместо увеличения дозы необходимо обучить пациента;

Развитие толерантности. При развитии толерантности к нитратам необходимо обеспечить «безнитратный промежуток», а не увеличивать дозу препарата;

Взаимодействия с другими ЛС, пищей, пищевыми добавками и др. Например, применение клопидогрела совместно с ингибиторами протонного насоса снижает антиагрегационный эффект первого;

Тяжелое течение заболевания. Неэффективность нитратов может говорить о развитии нестабильной стенокардии;

Неправильный диагноз. Например, нитроглицерин почти неэффективен при кардиалгиях некоронарного генеза;

Синдром отмены. Гипертонический криз, возникший вследствие отмены клонидина, как правило, не купируется стандартными средствами и требует назначения клонидина.

Иногда сбор фармакологического анамнеза способствует уточнению диагноза. Например, интерстициальный фиброз легких может быть вызван приемом амиодарона. Особую важность фармакологический анамнез имеет при обострениях хронических заболеваний, например гипогликемическая кома может быть вызвана относительной или абсолютной передозировкой гипогликемических препаратов.

Выполните задание для самостоятельной работы

Задание 3.7. Сформулируйте вопросы для сбора фармакологического анамнеза, соответствующие необходимой информации, у больного с заболеванием по вашей будущей специальности, при другой клинической ситуации, предложенной преподавателем, или курируемого больного. Заполните табл. 3.6.

Таблица 3.6. Фармакологический анамнез

Окончание табл. 3.6

Необходимая для фармакологического анамнеза информация

Сформулированный вопрос

Полученная от больного информация

Эффективность применяемых в настоящее время лекарственных средств

Переносимость (безопасность) применяемых в настоящее время лекарственных средств

Лекарственные средства, применяемые ранее в аналогичных ситуациях

Причины прекращения приема применяемых ранее лекарственных средств

Другие лекарственные средства, применяемые при сопутствующих заболеваниях или с другими целями (оральные контрацептивы)

Сопутствующая терапия «альтернативными» средствами: фитопрепаратами, гомеопатическими препаратами

Нежелательные лекарственные реакции при приеме ранее применяемых лекарственных средств

Отношение к алкоголю, курению, наркотикам

Что такое комплаентность (приверженность), и зачем врачу нужно ее улучшать?

Комплаентность - это приверженность пациента к лечению. От комплаентности может зависеть эффективность и безопасность назначаемых пациенту ЛП. Ни один современный и эффективный метод лечения не будет достаточно эффективен в случае, если пациент не

Понимает, зачем ему это нужно, и не выполняет предписаний. Следует помнить, что представление пациента о своей болезни и необходимом лечении формируется не только со слов врача, но и от знакомых, средств массовой информации, рекламы. Зачастую убедить пациента соблюдать врачебные рекомендации непросто. Кроме того, нужно учесть, что молодому врачу иногда сложнее повлиять на мнение старших по возрасту и социальному положению пациентов. Ниже приведены некоторые возможные причины несоблюдения пациентом врачебных рекомендаций: нежелание принимать лекарства (боится «употреблять химию», приверженность к народной медицине, экстрасенсам и пр.):

Пациент может опасаться нежелательных побочных реакций (даже ранее неизвестных);

Высокая стоимость лечения;

Неудобство, связанное с лечением (например, необходимость отказа от вождения автомобиля при приеме транквилизаторов, выраженный мочегонный эффект при приеме диуретиков, необходимость частого контроля коагулограммы при приеме антикоагулянтов);

Одновременный прием большого количества таблеток;

Длительность лечения, особенно когда пациент не ощущает наступления эффекта (например, многие пациенты, страдающие артериальной гипертензией, часто задают следующий вопрос: «Зачем мне всю жизнь принимать столько лекарств, если я не чувствую своего давления?») или когда нежелательная побочная реакция более «значима» для пациента, чем терапевтический эффект ЛС.

Выполните задание для самостоятельной работы

Задание 3.8. Проанализируйте случай из клинической практики. При ответе на вопросы используйте информацию из ТКФС «Бисопролол» (разделы «Фармакокинетика», «Режим дозирования») Государственного реестра лекарственных средств (www.regmed.ru).

Пациентке П. 82 лет по поводу артериальной гипертензии участковым терапевтом назначен бисопролол в дозе 5 мг 1 раз в сутки. При очередном визите к врачу в связи с высокими показателями АД рекомендовано увеличить дозу бисопролола до 10 мг в сутки. Однако в аптеке, куда обратилась пациентка, не оказалось таблеток бисопролола по 5 мг, в связи с чем она приобрела таблетки по 2,5 мг. Пациентка начала принимать бисопролол по 2,5 мг 4 раза в сутки,

объясняя это впоследствии неудобством одномоментного приема сразу 4 таблеток. Через 5 дней приема бисопролола в таком режиме у больной развивалась картина гипертонического криза, в связи с чем она госпитализирована в стационар.

1. Какая наиболее вероятная причина развития гипертонического криза у больной, исходя из фармакокинетики бисопролола?

2. Каковы причины низкой комплаентности у больной?

Для обеспечения максимальной комплаентности перед назначением ЛС пациенту врач должен:

Решить, сможет ли пациент по объективным причинам соблюдать план лечения и возможно ли обеспечить регулярный контроль терапии (например, визиты в поликлинику для контроля МНО при назначении варфарина могут быть затруднительны для пожилых пациентов, также пожилые пациенты могут забывать принимать ЛП и т.д.);

По возможности назначить ЛП 1-2 раза в сутки;

По возможности назначать комбинированные ЛП;

Назначать удобные средства «доставки» ЛС (например, ингаляторы);

Не менять без необходимости один ЛП на другой и не запутывать пациента разнообразными названиями препаратов, используйте МНН;

Предоставить четкую и понятную информацию по применению

ЛП (табл. 3.7).

Таблица 3.7. Информация, которую должен предоставить врач пациенту, при-

нимающему оральный антикоагулянт варфарин

Продолжение табл. 3.7

Как и когда применять лекарственное средство

Варфарин принимается всей суточной дозой в одно время, желательно в 17-19 ч. Запивают таблетки водой. Не рекомендуется прием вместе с пищей, можно принимать натощак.

Чтобы избежать развития осложнений, необходимая доза варфарина контролируется с помощью анализа крови, который называют МНО. Иногда в ответах лаборатории оно может обозначаться INR. В течение всего периода приема варфарина МНО должно находиться в интервале 2,0-3,0. Если МНО меньше 2,0, то свертываемость крови не снижена и возможны тромботические осложнения. Если МНО больше 4,0 - весьма реальны геморрагические осложнения. Повышение МНО от 2,5 до 4,0 говорит о необходимости снижения дозы препарата, но обычно не несет прямой угрозы. При некоторых заболеваниях необходимой верхней границей МНО считается 4,0-4,4.

Если нет возможности определять МНО, допустим контроль по протромбиновому времени (ПВ), однако этот способ значительно менее надежен. Для расчета дозы варфарина другие анализы крови не нужны. Для выявления побочного действия препарата периодически назначаются общий анализ крови, мочи и некоторые биохимические исследования. Подбор дозы варфарина.

Наиболее сложный и ответственный этап. Период подбора в среднем занимает от 1 до 2 нед, но в некоторых случаях увеличивается до 2 мес. В это время вам понадобится частое определение МНО, до 2-3 раз в неделю или ежедневно. Каждый раз, получив очередной результат исследования, ваш врач определит изменение дозы лекарства и дату очередного анализа. Если в нескольких анализах подряд МНО остается в интервале 2,0-2,5, это означает, что доза варфарина подобрана. Дальнейший контроль лечения будет значительно проще.

Продолжение табл. 3.7

Вопросы, на которые необходимо дать ответ больному

Конкретная информация для больного в отношении определенного лекарственного средства

Контроль дозы варфарина.

Если доза препарата подобрана, достаточен более редкий контроль - сначала 1 раз в 2 недели, затем 1 раз в месяц. Отдельно определяется периодичность дополнительных исследований. Необходимость внеочередного определения МНО может возникнуть в ряде случаев, о которых мы поговорим ниже. При любых сомнениях обратитесь к своему врачу за советом

Как долго следует применять лекарственное средство

Терапия варфарином длительная, иногда пожизненная. Для определения длительности терапии в некоторых случаях требуется проведение генетических исследований

Когда и какие положительные эффекты лекарственного средства можно ожидать

Лекарственный препарат призван защитить вашу жизнь, не допуская развития инфарктов, инсультов, тромбоэмболий

Возможные проблемы, которые могут возникнуть при применении лекарственного средства, и что делать при возникновении этих проблем

Применение варфарина может вызвать развитие кровотечений.

Немедленно сообщите врачу, если вы обнаружили:

Черный (дегтеобразный) цвет стула;

Розовый или красный цвет мочи;

Кровотечения из носа или десен (в том числе при чистке зубов);

Необычно обильные или продолжительные выделения при менструации;

Синяки или припухлости на теле, возникающие без видимой причины;

Любые значительные изменения в самочувствии и здоровье;

Появление кожных пятен на бедрах, брюшной стенке, молочных железах

Продолжение табл. 3.7

Вопросы, на которые необходимо дать ответ больному

Конкретная информация для больного в отношении определенного лекарственного средства

Какие пищевые продукты, напитки (в том числе и алкогольные) и другие лекарственные средства (в том числе и фитопрепараты) следует избегать

Что нужно избегать:

Применения других препаратов, влияющих на систему крови (в том числе содержащих ацетилсалициловую кислоту);

Применения препаратов, влияющих на метаболизм и выведение варфарина (антибиотики, противодиабетические препараты и др.). Если вам необходим прием какого-либо нового лекарственного препарата, обязательно обратитесь к врачу, сообщив ему, что вы принимаете варфарин;

Занятий травматичными видами спорта, где возможны удары, ушибы, падения;

Внутримышечных уколов. При амбулаторном лечении в большинстве случаев можно подобрать лекарства для приема внутрь;

Повторного приема лекарства в течение одного дня. Если вы не помните, принимали ли сегодня варфарин, пропустите прием;

Изменения питания.

Варфарин действует на свертываемость крови через витамин К, который в разных количествах содержится в пище. Не нужно избегать продуктов с высоким содержанием витамина К! Питание должно быть полноценным. Необходимо только следить, чтобы не было значительного изменения их доли в рационе, например в зависимости от сезона. Если вы значительно увеличите употребление пищевых продуктов, богатых витамином К на фоне подобранной стабильной дозы варфарина, это может сильно ослабить его действие и привести к тромбоэмболическим осложнениям. Максимальное количество витамина К (3000-6000 мкг/кг) содержится в темно-зеленых листовых овощах и травах (шпинате, петрушке, зеленой капусте), а в зеленом чае до 7000 мкг/кг; промежуточное количество (1000-2000 мкг/ кг) - в растениях с бледными листьями (белокочанной капусте, салате, брокколи, брюссельской капусте). Значительное количество витамина содержится в бобовых, майонезе (за счет растительных масел), зеленом чае.

Окончание табл. 3.7

Вопросы, на которые необходимо дать ответ больному

Конкретная информация для больного в отношении определенного лекарственного средства

Жиры и масла содержат разное количество витамина К (300-1000 мкг/кг), больше его в соевом, рапсовом, оливковом маслах. Содержание витамина К в молочных, мясных, хлебобулочных изделиях, грибах, овощах и фруктах, черном чае, кофе низкое (не более 100 мкг/кг). Регулярное употребление ягод и сока клюквы может усилить действие варфарина.

Небольшие дозы алкоголя при нормальной функции печени не влияют на терапию антикоагулянтами, однако к употреблению спиртного необходимо относиться осторожно.

Прием поливитаминов, содержащих витамин К, может ослабить действие варфарина

Что делать, если пропущена доза лекарственного средства

Принять препарат на следующий день

Где получить более подробную информацию о лекарственном средстве

Инструкция, ТКФС Государственного реестра ЛС

Выполните задание для самостоятельной работы

Задание 3.9. В соответствии с изложенным в табл. 3.7 планом сформулируйте рекомендации для больного по применению какого-либо ЛП. Используйте инструкцию по медицинскому применению ЛП и ТКФС.

В США инструкции ЛП для пациентов разработаны Национальным институтом здоровья и находятся в свободном доступе в Интернете: http://www.nlm.nih.gov/medlineplus/druginformation.ritml

Существуют ли дополнительные методы улучшения комплаентности?

Эффективными методами улучшения комплаентности также считаются:

Ведение дневников самонаблюдения;

Разработка для пациента индивидуального плана лечения;

Посещение школ пациентов с различными заболеваниями. Очень важно, чтобы пациент не только аккуратно осуществлял

прием ЛС, но и мог адекватно самостоятельно оценивать свое состояние, принимать необходимые меры и вовремя обращаться к врачу. Для этого полезно ведение дневника самонаблюдения (табл. 3.8), где пациент фиксировал бы свои жалобы, ключевые лабораторноинструментальные показатели (уровень АД при артериальной гипертензии, объем диуреза при сердечной недостаточности, температуру тела при инфекционном заболевании, уровень глюкозы при сахарном диабете и др.), а также дополнительный прием ЛС. Подобный дневник позволит врачу более адекватно оценить динамику состояния и эффективность проводимой терапии, а также повысит приверженность пациента к лечению.

Таблица 3.8. Дневник самонаблюдения пациента с бронхиальной астмой

Выполните задание для самостоятельной работы

Задание 3.10. Используя в качестве примера табл. 3.8, разработайте дневник самонаблюдения для пациента по профилю вашей будущей специальности.

Еще одним важным компонентом улучшения комплаентности считается составление индивидуального плана лечения, где пациенту будут даны рекомендации о том, в каких случаях необходимо самостоятельно изменить прием препаратов, когда следует немедленно обратиться к врачу и что делать в экстренных ситуациях (табл. 3.9).

Таблица 3.9. Индивидуальный план лечения больного бронхиальной астмой

I. Ваше основное лечение:

Ежедневно принимайте:

1. 25 мкг салметерола и 125 мкг флутиказона по 2 вдоха утром и вечером.

2. Зафирлукаст по 10 мг утром и вечером внутрь, за 1 ч до еды или 2 ч после еды.

3. Перед физической нагрузкой примите: сальбутамол 0,2 мг 1-2 вдоха за 15-30 мин. При возникновении симптомов «по требованию» примите: сальбутамол 0,2 мг 1-2 вдоха.

II. Когда необходимо увеличить лечение?

Ответьте на вопросы, проанализировав свое состояние за последнюю неделю:

Появлялись симптомы (одышка, кашель, свисты, стеснение в груди и

пр.) более 2 раз в день?

Из-за астмы вы просыпались ночью?

Вам приходилось использовать ингалятор «по требованию» более 2 раз? Ваша физическая активность снизилась из-за астмы?

Уровень ПСВ стал ниже_л/мин?

Если вы ответили «ДА» 3 раза или больше, вам необходимо увеличить лечение.

Ежедневно принимайте:

1. 25 мкг салметерола и 250 мкг флутиказона по 2 вдоха утром и вечером.

2. Зафирлукаст по 20 мг утром и вечером внутрь, за 1 ч до еды или 2 ч после еды.

3. Оценивайте свое состояние ежедневно, придерживайтесь этой схемы лечения в течение недели.

III. Когда идти на прием к врачу?

Если в течение недели улучшения не наступает, запишитесь на прием:

_(указать телефон)_(регистратура)

Врач: Ф.И.О._

IV. Экстренная ситуация.

У вас появилась тяжелая одышка, вы можете произнести только короткие предложения.

У вас тяжелый приступ астмы и вы напуганы.

Вы используете ингалятор «по требованию» каждые 4 ч, а улучшение не наступает.

Сделайте 2-4 вдоха сальбутамола. Примите 20 мг метилпреднизолона внутрь. Обратитесь за помощью «03».

Продолжайте ингаляции сальбутамола по 2-4 вдоха каждые 20-30 мин до приезда врача.

Выполните задание для самостоятельной работы

Задание 3.11. Используя приведенный ниже пример индивидуального плана лечения больного бронхиальной астмой (см. табл. 3.9 из Global Initiative for Asthma, GINA, 2006), разработайте индивидуальный план лечения пациента по вашей будущей специальности.

Еще один способ повышения комплаентности и мотивации больных к лечению - это создание специальных школ здоровья для больных определенными заболеваниями (школа здоровья для больных сахарным диабетом, артериальной гипертензией, бронхиальной астмой, хроническим вирусным гепатитом, язвенной болезнью, остеопорозом, миопией и пр.). Основные задачи школ:

1. Дать пациенту представление о его заболевании, рассказать о современных возможностях лечения.

2. Дать возможность пациенту оценить тяжесть своего состояния и адекватность проводимого лечения.

3. Научить пациента самостоятельно распознавать приближающееся ухудшение и предотвращать его.

4. Научить пациента самостоятельной помощи при развившемся обострении.

5. Объяснить назначение приборов индивидуального пользования (например, спейсера, пикфлоуметра, глюкометра, тонометра и др.).

6. Научить ведению дневника здоровья.

8. Сформировать у пациента навыки «ответственного самолечения». Под самолечением (по определению экспертов ВОЗ) понимают «разумное применение самим пациентом ЛС, находящихся в свободной продаже, в целях профилактики или лечения легких расстройств здоровья до оказания профессиональной врачебной помощи». Следует информировать пациента, что «самолечение может быть использовано лишь при ограниченном числе недомоганий» и должно быть ограничено во времени. В числе ЛС, которые могут использоваться при самолечении, должны быть только безрецептурные ЛС, применяемые больным строго по инструкции по применению ЛП, т.е. листку-вкладышу: показания, противопоказания, дозы, пути введения, кратность применения, длительность применения и т.д.

Пример плана занятий в школе больных сахарным диабетом второго типа, проводившихся на базе клиники эндокринологии Московской медицинской академии им. И.М. Сеченова (табл. 3.10).

Таблица 3.10. План занятий в школе больных сахарным диабетом второго типа Занятие 1. Тема «Что такое СД»

Основные понятия диабетологии (нормальный, высокий и низкий уровни сахара в крови, почечный порог комы). Цели лечения.

Симптомы СД и причина их появления.

«Четыре кита» комплексной терапии СД второго типа. Занятие 2. Тема «Самоконтроль»

Что входит в понятие самоконтроля и почему он необходим.

Способы самоконтроля сахара в крови, моче, ацетона в моче.

Обучение практическим навыкам проведения самоконтроля.

Ведение «Дневника больного СД второго типа». Занятия 3, 4. Тема «Основные принципы диетотерапии при СД второго типа»

Понятие об энергетическом балансе здорового человека и пациента с

СД второго типа.

Основные составляющие пищи и понятие о калорийности. Способы уменьшения калорийности суточного рациона. Углеводы как основной источник энергии, классификация углеводов. Система «хлебных единиц», взаимозаменяемость продуктов, содержащих углеводы. Пищевой «светофор».

Употребление сахарозаменителей и подсластителей. Как быть с алкоголем? Занятие 5. Тема «Физические нагрузки. Гипогликемия»

Часть I. Физические нагрузки:

Необходимость расширения физической активности при СД второго типа;

Основные правила дозировки физических нагрузок;

Поведение перед началом и в процессе физической нагрузки. Часть II. Гипогликемия:

Что такое гипогликемия;

Причины развития гипогликемии;

Симптомы легкой и тяжелой гипогликемии;

Что делать при возникновении симптомов гипогликемии.

Окончание табл. 310

Выполните задание для самостоятельной работы

Задание 3.12. Используя приведенный пример, разработайте план занятий в школе пациента из вашей будущей специальности.

Занятие 6. Тема «Поздние осложнения СД: ретинопатия, нефропатия, нейропатия»

Обсуждение наиболее часто встречающихся осложнений СД и причин их возникновения, введение терминов «ретинопатия», «нефропатия», «нейропатия».

Ретинопатия: стадии, симптомы, профилактика, лечение. Нефропатия: стадии, симптомы, профилактика, лечение. Нейропатия: формы, симптомы, профилактика, лечение. Занятие 7. Тема «Правила ухода за ногами» Причины поражения ног при СД.

Профилактика поражения ног (правила «можно-нельзя»).

Обучение навыкам осмотра обуви и поведению при возникновении

повреждений ног. Занятие 8. Тема «Артериальная гипертензия, атеросклероз и СД второго типа»

Часть I. Артериальная гипертензия:

Что такое артериальное давление;

Нормальный уровень АД, причины и механизмы повышения АД;

Симптомы повышенного АД;

Опасности повышенного АД;

Принципы современной антигипертензивной терапии. Часть II. Атеросклероз:

Что такое атеросклероз и механизмы его развития;

Нормальный уровень холестерина в крови;

Проявления атеросклероза;

Принципы современной гиполипидемической терапии. Занятие 9. Тема «Инсулинотерапия при СД второго типа»

Причины назначения инсулина при СД второго типа. Механизм действия инсулина.

Виды препаратов инсулина и системы для введения инсулина. Схемы инсулинотерапии. Обучение технике инъекций.

Выполните задание для самостоятельной работы

Задание 3.13. Проанализируйте случай из клинической практики. Пациент К. 58 лет наблюдается у кардиолога по месту жительства с диагнозом: ИБС, стенокардия напряжения II ФК-класса. Постоянная форма мерцательной аритмии. Артериальная гипертензия 3 степени, очень высокий риск. Диффузно-узловой зоб, эутиреоз. Язвенная болезнь желудка, ремиссия. В связи с наличием у больного постоянной формы мерцательной аритмии для профилактики тромбоэмболических осложнений кардиологом назначен варфарин, под контролем МНО подобрана доза варфарина 7,5 мг в сутки, на фоне приема которой МНО составляло 2,37-2,5. Болей в животе, геморрагий не отмечал. Через 3 мес после начала приема варфарина больной доставлен бригадой скорой помощи из общественного места в стационар в связи с развившимися болями в прекардиальной области (общая продолжительность приступа 10 мин), купированными после 4-кратного применения спрея нитроглицерина (со слов больного). При поступлении данных за острый коронарный синдром не было, клиническая картина геморрагического синдрома отсутствовала. Больной осмотрен врачамитерапевтами приемного отделения и дежурным терапевтом. Назначено обследование (клинический анализ крови, общий анализ мочи, реакция Вассермана, ЭКГ, биохимический анализ крови, МНО), лечение (изосорбида динитрат, варфарин - 7,5 мг в сутки, метопролол, эналаприл, индапамид). Однако больной в этот же день самостоятельно покинул отделение после осмотра дежурного терапевта, в связи с чем назначенное обследование и лечение начато не было. Через месяц больной доставлен бригадой скорой медицинской помощи в стационар с явлениями геморрагического синдрома: подкожные кровоизлияния, субконъюнктивальное кровоизлияние, гематурия, МНО при этом составляло 9,8. Со слов больного, в течение последнего месяца принимал варфарин в прежней дозе (7,5 мг в сутки), однако МНО не контролировал (пропустил плановое измерение МНО), так как находился за городом, к врачам не обращался. Также больной утверждает, что в течение последнего месяца злоупотреблял алкоголем.

В стационаре варфарин был отменен на фоне переливания свежезамороженной плазмы, применения препарата витамина К. Геморрагический синдром был купирован, МНО снизилось и при выписке составляло 1,12. Больной выписан в удовлетворительном состоянии с рекомендациями не принимать варфарин в связи с невозможностью контролировать МНО.

1. Какие действия больного привели к развитию геморрагического синдрома?

2. Какие причины низкого комплаенса у больного?

3. Как можно избежать развития геморрагического синдрома у больного?

Клиническая фармакология. Общие вопросы клинической фармакологии. Практикум: учебное пособие. Сычев Д.А., Долженкова Л.С., Прозорова В.К. и др. / Под ред. В.Г. Кукеса. 2013. - 224 с.: ил.

Выберите правильное утверждение: а)биодоступность-количество ЛС,поступающее в системный кровоток,выраженное в процентах от введенной дозы,б)биодоступность определяется величиной адсорбции ЛС в ЖКТ и выраженностью эффекта первого прохождения через печень.в)биодоступность определяют по формуле: F = AUC (в/м или внутрь)/AUC (в/в).г)биодоступность ЛС при внутримышечном введении определяется степенью его всасывания и биотрансформации в организме.
Ответ: а, б, в

2.
Ответ: Атровент

3.

Ответ: а, д

4.

Ответ:

5.
Ответ:

6.

д) Ксилит
Ответ: а,в

7.

Ответ: а,б,д

8.
Ответ:

9. поступил с острым инфарктом миокарда,возникшим 5ч назад.Назначения: анаприлин 20 мг 4 раза в сутки внутрь, гепарин в/в капельно по 10 000 ЕД каждые 4 часа.При этом удалось достигнуть увеличения времени свертывания крови до 18-23 минуты. На следующий день у б-ного диагностирована правосторонняя нижнедолевая пневмония.Назначена натриевая соль бензилпенициллина (по 1000 000 ЕД каждые 4 ч) в/в.Через 4 ч время свертывания крови составило 8 минут. Какова ваша тактика?
Ответ:

10.

11.
Ответ: Вит.В12 в дозе 500 мкг/сут через день, фолиевая кислота в дозе 1,5 мг/сут, сульфат железа(80 мг Fe2+) 1 раз в сутки

12.

Ответ: Вит.С

13.

Ответ: Церебролизин

14.
аллергию (на бутадион, гепарин, метиндол, пенициллин, теофиллин).В стаци-онаре б-ной назначены реопирин по 5 мл в/м 1 раз в сутки,гидрокортизона гемисукцинат по 100 мг в полость коленных суставов,тавегил по 0,001 г 2р.в сутки.Через 3 дня у б-ной появились зудящие эритематозные высыпания на коже туловища.Какова наиболее вероятная причина
ухудшения состояния?
Ответ:

15.



Ответ: а,б,д,е,з,и

16.
Ответ: Через несколько месяцев

17.


Ответ: а,б,в,д,е

18.
Ответ: а,б,в,г,д,ж,з

19.
Ответ:

20.
Ответ: Ципрофлоксацин

21. Феномен первого прохождения ЛС через печень зависит от:а) кровоснабжения печени, б) связывания ЛС с белком, в) активности ферментов гепатоцитов, г) уровня экскреции ЛС,д) быстроты всасывания
Ответ: а, в

22. Препараты, влияющие на микросомальные ферменты печени:индукторы микросомальных ферментов печени: а) пенициллин, б) нитроглицерин, в) фенобарбитал, г) фуросемид, д) бутадион, е) кортизол, ж) пропранолол, з) циметидин, и) левомицетин, к) дифенин
Ответ: в,д

23. поступила в отделение с ж-ми на боли в правой молочной железе, повышение Т.до 39,5 С.Заболела 3 дня назад,на 10-й день после родов. При поступлении в отделение в верхненаружном квадранте правой молочной железы обнаружена гиперемия кожи,массивный инфильтрат с флуктуацией в центре.Диагноз:острый правосторонний мастит.Б-ная оперирована. Взят посев отделяемого раны.Определите антибиотик первого выбора
Ответ: Цефазолин

24.

Ответ: Анафилактическая реакция

25.

Ответ: Левомицетин

26.
Ответ:

27.
Ответ: Бигуаниды

28.

Ответ: Гипотония, головокружение.

29.

Оцените действия врача.

30. У больного Д., 53 лет, диагноз: ИБС, стабильная стенокардия??? ФК, постинфарктный кардиосклероз, мерцательная аритмия,ХНК??Б ст. Принимал строфантин, дигоксин, фуросемид, панангин в средних терапевтических дозах. Неожиданно у больного повысилась температура до 38,4°С, появилась кашель, одышка, крепитация в легких справа. На рентгенограмме легких справа в нижней доле определяется участок инфильтрации. К лечению добавлены гентамицин, сульфокамфокаин,супрастин.

Ответ:

31.

Ответ: Фентоламин.

32.

Ответ: г,д

33.

Ответ:

34.
Ответ:

35.
Ответ: Эналаприл.

36.

Ответ: а,б,г

37.

Ответ:

38.
Ответ:

39.
Ответ:

40.

Ответ: все перечисленное

41. Препараты, влияющие на микросомальные ферменты печени: ингибиторы микросомальных ферментов печени:
а) пенициллин, б) нитроглицерин,в) фенобарбитал,
г) фуросемид, д) бутадион, е) кортизол, ж)пропранолол,
з) циметидин, и) левомицетин, к) дифенин
Ответ: з,и

42.
Ответ: Через 7-14 дней

43. Укажите комбинацию препаратов, приводящую к возникновению конкуренции за связывание с белком, что может обусловить опасное повышение содержания свободной фракции одного из препаратов в крови и появление симптомов его передозировки:
Ответ: неодикумарин и бутадион

44. Выберите ЛС с узким терапевтическим диапазоном:
а) пенициллины, б) противосудорожные средства,
в) антиаритмические средства,г) дигоксин, д) метотрексат, е) теофиллин, ж) циклоспорин, з) макролиды
Ответ: б,в,г,д,е,ж

45. Укажите комбинации препаратов, при которых вследствие конкуренции за связь с белком происходит увеличение концентрации в плазме крови свободной фракции одного из них: а.строфантин и мисклерон, б.дигитоксин и мисклерон, в.неодикумарин и бутадион, г.нифедипин и гидрохлортиазид
Ответ: б,в

46. выявлены частая желудочковая экстрасистолия и пароксизмы мерцательной аритмии.ЧСС 74 в мин,АД 140/80 мм.рт.ст.Последние 3г.беспокоят приступы стенокардии напряжения и покоя.Лечение проводили кордароном.Учитывая побочные эффекты назначенного препарата,выберите ЛС для дальнейшего лечения б-ного: а) Хинидин,
б) Боннекор, в) Этацизин,

Ответ: а,б

47. Известно,что при комбинации хинидина и дигоксина часто наблюдается гликозидная интоксикация.С чем она связана? Фармакодинамическое взаимодействие:
Ответ: синергизм

48. Известно,что при комбинации хинидина и дигоксина наблюдается гликозидная интоксикация.С чем она связана? Фармакокинетическое взаимодействие, влияние хинидина на:
Ответ: связь с белком

49. Критические периоды внутриутробного развития:
а. период предимплантационного развития (1 нед)
б. стадия эмбриогенеза заканчивается к 8 нед.
в. стадия эмбриогенеза заканчивается к 8 мес.
г. период непосредственно перед родами
Ответ: а,б,г

50. Выберите из нижеуказанных препаратов,обладающие перечисленными свойствами: Противомикробные препараты,применение которых практически безопасно при беременности: а.сульфаниламды,в том числе бисептол,
б.аминогликозиды,тетрациклины,рифампицины,метронидазол (в 1триместр беременности), в. пенициллины, цефалоспорины, эритромицин,линкомицин,фузидин, г.антимикотические средства,противоопухолевые
антибиотики.
Ответ: в

51. Метронидазол назначен кормящей матери, укажите побочные эффекты:
а.повышение возбудимости,тахикардия, б.угнетение аппетита,рвота, в.угнетение ЦНС,дыхания,снижение массы тела, г.увеличение секреции пролактина,нагрубание молочных желез, д.гипоплазия надпочечников, нарушение обмена веществ,повышение риска развития билирубиновой энцефалопатии, е.геморрагии,нарушение дыхания,ацидоз, ж.угнетение кроветворения,анемия, гипотрофия, дисбактериоз.
Ответ: б

52. Антимикробные препараты первого выбора у новорожденных: а.бензилпенициллин,оксациллин,карбенициллин,гентамицин,амикацин,б.бензилпенициллин,оксациллин,бициллины,цефазолин,цефотаксим,эритромицин,линкомицин,нистатин, в.карбенициллин,гентамицин,сизомицин,амикацин, тобрамицин,цепорин (при неэффективности цефалоспоринов первого поколения), г.эритромицин,линкомицин,нистатин,леворин,карбенициллин,
гентамицин, сизомицин
Ответ: б

53.
Ответ:

54. Основные особенности фармакокинетики ЛС у лиц пожилого возраста:
а.снижение скорости абсорбции, б.ускорение абсорбции, в.снижение скорости распределения, г.ускорение распределения, д.уменьшение связывания ЛС с белками плазмы, е.увеличение связывания ЛС с белками плазмы, ж.замедление метаболизма, з.ускорение метаболизма,
и.замедление выведения ЛС, к. ускорение выведения ЛС.
Ответ: а,в,д,ж,и

55.
Ответ: б,в,г

56. Укажите побочные эффекты бета-адреноблокаторов: а)брадикардия, б)артериальная гипотензия, в)бронхоспазм, г)тахикардия, д)нарушение функций щитовидной железы,
е) перемежающаяся хромота, ж) АВ-блокада
Ответ: а,б,в,е,ж

57.
состояния: а)Естественное течение заболевания, б)Развитие толерантности к нитратам, в)Синдром межкоронарного обкрадывания, г) Возникновение синдрома рикошета д) явления идиосинкразии
Ответ: а,б

58. Укажите побочные эффекты амиодарона: а) брадикардия, б).артериальная гипотензия, в) бронхоспазм,г)тахикардия, д) нарушение функций щитовидной железы, е) перемежающаяся хромота, ж) АВ-блокада
Ответ: а,в,д,ж

59. Как изменится ваша антиангинальная терапия,если у б-ного на фоне терапии нитратами возникнет инсульт головного мозга?
Ответ: отмена нитратов и назначение антиангинального препарата другой группы

60. Какие гипотензивные препараты считают наиболее безопасными для пожилых больных: а)бета-адреноблокаторы, б)ганглиоблокаторы, в)симпатолитики, г)блокаторы медленных кальциевых каналов, д)тиазидные
диуретики, е) ингибиторы АПФ.
Ответ: г,д

61. Схема лечения кордароном:
Ответ: по схеме, предполагающей постепенное снижение дозы с 600 мг до 200 мг в сутки

62. Как влияют ингибиторы МАО (антидепрессанты) на прессорный эффект адреностимуляторов прямого и непрямого действия?
Ответ: усиливают действие

63. Небензодиазепиновый» агонист бензодиазепиновых рецепторов:
Ответ: Золпидем

64. Снотворное средство — соединение алифатического ряда:
Ответ: Хлоралгидрат

65.

Ответ: А (б)

66. Протамина сульфат назначают при передозировке:
Ответ: Гепарина

67. Какой способ детоксикации наиболее эффективен при отравлении веществами, связывающими с белками и липидами крови?
Ответ: Гемосорбция

68. Принцип действия налоксона при остром отравлении морфином:
Ответ: Препятствует действию морфина на опиоидные рецепторы

69. Укажите препараты,обладающие свойствами антиоксидантов: а)верапамил б) вит.А, в) вит.К, г) вит.С, д) вит.Е, е)селен, ж) карнозин, з) доксициклин
Ответ: б,г,д,е,ж

70. Какие эффекты характерны для нейролептиков?
а) антипсихотический, б) седативный, в) противорвотный
Ответ: а,б,в

71. У Б-ой 64л.развился острый приступ закрытоугольной глаукомы с сильными болями в правом глазу с иррадиацией в голову. Появились тошнота и рвота,одышка,выявлены признаки гипертонического криза 2 типа с ЧСС 62
в мин.АД 200/140 мм.рт.ст.Б-ная в течение многих лет страдает гипертонической болезнью.В легких большое кол-во влажных мелкопузырчатых хрипов.Какие диуретики показаны больной? а. Клопамид, б.Верошпирон,в.Гипотиазид, г.Фуросемид в/в, д. Диакарб:
Ответ: г,д

72. в течение 15 лет страдает сахарным диабетом,по поводу которого получает инсулин по 70 ЕД/сут,что поддерживает уровень гликемии в пределах 7,5-8,6ммоль/л.В последнее время стало повышаться АД до
170/90-180/100 мм.рт.ст.в связи с чем лечащий врач назначил обзидан в суточной дозе 120 мг.Какие побочные эффекты следует ожидать при данной комбинации ЛС? а.Гипергликемия вплоть до комы, б.Сердечная недостаточность, в.Гипогликемия вплоть до комы, г.Ортостатическая гипотензия, д. Гипертензия
Ответ: б,в

73. , страдающему гормонально-зависимой бронхиальной астмой,назначили преднизолон (по 5 мг ежедневно),сальбутамол (ингаляция 2 доз аэрозоля 4р.в сутки).В связи с проявлениями судорожного синдрома (в анамнезе черепно-мозговая травма) был назначен фенобарбитал.Через неделю у б-го развилось обострение бронхиальной астмы.С чем это связано?
А. Фенобарбитал ускорил биотрансформацию: а.сальбутамола, б.преднизолона, Б.Фенобарбитал ускорил экскрецию: а.сальбутамола, б.преднизолона, В.Фенобарбитал замедлил экскрецию: а.сальбутамола, б.преднизолона, Г.Фенобарбитал замедлил биотрансформацию:а.сальбутамола,б.преднизолона
Ответ: А (б)

74. страдает ИБС,стенокардией напряжения III ФК.ЧСС 90 в мин,АД 150/80мм.рт.ст.В анамнезе-хронический бронхит с бронхоспастическим синдромом в стадии ремиссии.Жировая дистрофия печени.Укажите группы ЛС (второй этап выбора лекарственной терапии),оптимальные для антиангинальной терапии. а.Нитраты и верапамил, б. Нитраты и атенолол
в) Нитраты и анаприлин, г) Нитраты и нифедипин,
д) Нифедипин и амиодарон
Ответ: а

75. по поводу стенокардии принимает нитросорбид по 10мг 4р.в сутки ЧСС 80 в ми.АД 140/80мм.рт.ст.Через 1 мес.после начала терапии вновь участились приступы стенокардии.Каковы возможные причины ухудшения
состояния: а)Естественное течение заболевания, б)Развитие толерантности к нитратам, в)Синдром межкоронарного обкрадывания, г) Возникновение синдрома рикошета, д) явления идиосинкразии
Ответ: а,б

76. отмечаются приступы стенокардии при умеренных физических нагрузках.В анамнезе коллаптоидное состояние после однократного приема нитроглицерина сублингвально (с тех пор б-ной нитроглицерин не принимал). Сопутствующие заболевания-ГБ (рабочий уровень АД 160/100мм.рт.
ст.,гипофункция щитовидной железы.На момент осмотра АД 190/100мм.рт.ст.,ЧСС 72 в минуту.Больному противопоказаны:
Ответ: Амиодарон

77. по поводу артериальной гипертензии 2степени получает 0,000075г клофелина 4р.в сутки.В связи с развитием сенильной депрессии назначен мелипрамин.Через 3 дня после назначения мелипрамина у б-ного возник гипертонический криз.Накануне б-ной не принимал клофелин.Каковы возможные причины ухудшения состояния: а)Следствие естественного течения заболевания, б)Следствие гипертензивного эффекта мелипрамина, в)Следствие неблагоприятного взаимодействия ЛС, г)Следствие возможного прекращения приема ЛС б-ным и развития синдрома отмены.
Ответ: б,в,г

78. по поводу гипертонического криза вводили натрия нитропруссид в/в капельно в больших дозах (со скоростью 8 мкг/мин). Появились одышка, акроцианоз,давящие боли за грудиной,мышечные подергивания.Какова причина ухудшения состояния б-ного?
Ответ: Токсическое действие цианидов

79. выявлены частая желудочковая экстрасистолия и пароксизмы мерцательной аритмии.ЧСС 74 в мин,АД 140/80 мм.рт.ст.Последние 3г.беспокоят приступы стенокардии напряжения и покоя.Лечение проводили кордароном.Учитывая побочные эффекты назначенного препарата,выберите ЛС
для дальнейшего лечения б-ного: а) Хинидин, б) Боннекор, в) Этацизин,
г) Мекситил, д) Верапамил, е) Пропранолол
Ответ: а,б

80. пароксизмальная суправентрикулярная тахикардия на фоне синдрома WPW.Для купирования приступа выбран аймалин.Определите оптимальную схему лечения выбранным препаратом: а)1 мг/кг в/в в течение более10 мин,при необходимости повторить через 30 мин, б)50 мг в/в в тече-
ние 3-5 мин в 10 мл 5% раствора глюкозы или изотонического раствора NaCl или в/м, в) 0,5-1 г в/в каждые 2 мин вводят по 0,1-0,2 г или в/м
г) после парентерального введения назначить внутрь по 100мг 4-5 раз в сутки, поддерживающая доза 50 мг 3-4 раза в сутки
Ответ: а,г

81. У б-ной 28л.с диагнозом СКВ на фоне ХПН появились отеки голеней,увеличение печени.При эхокардиографическом исследовании определяется снижение сердечного выброса. ЧСС 95 в мин,АД 170/100 мм.рт.ст.Какие сердечные гликозиды показаны больной?
Ответ: Дигитоксин

82. У б-ной 28л.с диагнозом СКВ на фоне ХПН появились отеки голеней,увеличение печени.При эхокардиографическом исследовании определяется снижение сердечного выброса.ЧСС 95 в мин, АД 170/100 мм.рт.ст. Больная принимает дигитоксин.В связи с появлением судорожного синдрома дополнительно назначен фенобарбитал (0,3 г/сут).Когда возникнут изменения в состоянии б-ной при наличии влияния?
Ответ: Через 7-14 дней

83. 57 лет по поводу постинфарктного атерокардиосклероза,застойной сердечной недостаточности 2 степени получает 40 мг фуросемида в/в и 300 мг
верошпирона внутрь.Какую диуретическую терапию Вы назначите б-ному в случае рефрактерности?
Ответ: Фуросемид 80 мг в/в и спиронолактон 300 мг внутрь

84. страдает неатопической бронхиальной астмой,сопровождающейся обильной бронхореей.Пульс 62 в мин.АД 140/80 мм.рт.ст.Какие препараты более предпочтительны?
Ответ: Атровент

85. упорно рецидивирующий синдром бронхиальной обстукции со сниженной чувствительностью к холино и адренотропным средствам.Бронхиальной астмой страдает более 10 лет.Что можно назначить для уменьшения частоты и тяжести приступов бронхиальной астмы: а)Ингаляции бета
2-адреностимуляторов более 6 раз в сутки, б) Ингаляция м-холиноблокатора, в) Введение адреналина п/к в большей дозе,чем обычно,для снятия бронхоспазма, г)Эуфиллин в/в, д) Ингаляционные глюкокортикоиды.
Ответ: г,д

86. поступил с ж-ми на изжогу,боли в эпигастральной области натощак, купируемые приемом натрия гидрокарбоната.При ФЭГДС выявлена язва (0,5см в диаметре) в ампуле 12 п.к.РН-метрия желудочного сока: кислотообра-
зующая функция средней интенсивности с низкими щелочными резервами,холинергический тип рецепции. Диагноз: язвенная болезнь 12 п.к.в стадии обострения. Выберите наиболее эффективное и безопасное ЛС и определите режим его дозирования:
Ответ: Пирензепин до еды по 0,05 г 3 раза в сутки в течение 2дней, затем по 0,05 г 2 раза в сутки

87. выявлена дискенезия желчного пузыря по гипертоническому типу. Выберите оптимальный вариант лечения.
Ответ: Но-шпа по 1-2 табл.3 раза в сутки, отвар бессмертника по 1/2 стакана за 30 мин до еды

88. страдает хрон.холецистопанкреатитом в течение 5 лет.За последнюю неделю после нарушения диеты отмечает усиление болей в правом по дреберье,тошноту,горечь во рту.Выберите наиболее эффективные желчегонные средства, обладающие одновременно противомикробной активностью:
а) Аллохол, б) Холензим, в) Никодин, г) Отвар пижмы,д) Ксилит
Ответ: а,в

89. с суицидальной целью выпила 20 таблеток феназепама.Через 2 ч после приема препарата доставлена в стационар. Б-я в сознании,но резко заторможена.Проведено промывание желудка.Выберите наиболее оптимальные слабительные средства: а) Глауберова соль, б) Сульфат магния, в) Экстракт коры крушины, г) Бисакодил,
д) Касторовое масло, е) Морская капуста, ж) Вазелиновое масло
Ответ: а,б,д

90. 46 лет поступил в отделение кардиореанимации с острым трансмуральным инфарктом миокарда, возникшим около 5 ч назад.Назначения:анаприлин 20 мг 4 раза в сутки внутрь, гепарин в/в капельно по 10 000 ЕД каждые 4 часа.При этом удалось достигнуть увеличения времени свертывания крови до 18-23 минуты. На 4-й день у б-ного выявлена микрогема турия (22 эритроцита в поле зрения). Какова ваша тактика?
Ответ: Снизить дозу гепарина вплоть до времени свертывания крови не менее 10-12 мин

91. поступил с острым инфарктом миокарда,возникшим 5ч назад.Назначения: анаприлин 20 мг 4 раза в сутки внутрь, гепарин в/в капельно по 10 000 ЕД каждые 4 часа.При этом удалось достигнуть увеличения времени свертывания крови до 18-23 минуты. На следующий день у б-ного диагностирова на правосторонняя нижнедолевая пневмония.Назначена натриевая соль бензилпенициллина (по 1000 000 ЕД каждые 4 ч) в/в.Через 4 ч время свертывания крови составило 8 минут. Какова ваша тактика?
Ответ: Изменить путь введения пенициллина

92. проведена радикальная операция по поводу рака желудка. На 4 сутки после операции при исследовании коагулограммы выявлены гиперкоагуляция и снижение фибринолитической активности крови.Целесообразно ли назначение антикоагулянтов?
Ответ: Антикоагулянты показаны,однако необходим тщательный контроль для предупреждения геморрагического синдрома

93. поступила в стац.с жал-ми на резкую слабость, одышку при ходьбе. При обследовании в анализе крови выявлена анемия (гемоглобин-56 г/л), цветной показатель 1,2,при осмотре языка-глоссит.В пунктате костного мозга выявлен мегалобластный тип кроветворения.Концентрация железа в сыворотке крови в пределах нормы.Диагноз: В 12-дефицитная анамия.Выберите наиболее оптимальный вариант лечения.
Ответ: Вит.В12 в дозе 500 мкг/сут через день, фолиевая кислота в дозе 1,5 мг/сут, сульфат железа(80 мгFe2+) 1 раз в сутки

94. после переохлаждения возникли озноб,повышение Т.тела до 38,6С, кашель с отделением слизисто-гнойной мокроты,боли в правой половине грудной клетки.Клинически и рентгенологически установлен диагноз правосторонней нижнедолевой пневмонии.Б-ной назначено лечение:цефазолин по 0,5 г 2р.в сутки вм,гемодез 400 мл в/в капельно, отхаркивающая микстура по 1ст.л.6р.в сутки.Выберите антиоксидантный препарат,наиболее
эффективно влияющий на процессы свободнорадикального окисления в легких,который следует добавить к проводимой терапии
Ответ: Вит.С

95. по поводу острого нарушения мозгового кровообращения по ишемическому типу давностью 12ч получает реополиглюкин по 400мл в/в капельно
1р.в сутки.Выберите наиболее эффективный в этой ситуации препарат,обладающий антиоксидантными свойствами
Ответ: Церебролизин

96. в течение 5 лет страдает деформирующим остеоартрозом нижних конечностей с выраженными синовитами. В анамнезе отмечает лекарственную
аллергию (на бутадион,гепарин,метиндол,пенициллин,теофиллин).В стаци-онаре б-ной назначены реопирин по 5 мл в/м 1 раз в сутки,гидрокортизона гемисукцинат по 100 мг в полость коленных суставов,тавегил по 0,001 г 2р.в сутки.Через 3 дня у б-ной появились зудящие эритематозные высыпания на коже туловища.Какова наиболее вероятная причина ухудшения состояния?
Ответ: Лекарственная аллергическая реакция

97. подтвержден диагноз ревматоидного артрита.Какие базисные средства для лечения ревматоидного артрита вы можете назначить: а)4,7-хлорхинолоновые препараты (делагил), б) Цитостатики (азатиоприн, циклофосфан и др.), в) Глюкокортикоиды (преднизолон), г) НПВС,
д) Препараты золота (кризанол), е) Салазопиридазин,
ж) Антибиотики (тетрациклины), з) D-пеницилламин,
и) Иммуномодуляторы (левамизол)
Ответ: а,б,д,е,з,и

98. Пациенту с ревматоидным артритом был назначен метотрексат. Как скоро проявится действие метотрексата?
Ответ: Через несколько месяцев

99. с ревматоидным артритом был назначен метотрексат. Какие вы предпримите меры по контролю за безопасностью фармакотерапии метотрексатом у данного пациента: а) Еженедельное проведение общего анализа крови
(лучше 2р.в неделю), б) Проведение анализа крови с определением количества тромбоцитов каждые 3-4 нед,
в) Проведение общего анализа мочи, г)Определение содержания мочевой кислоты, д) Проведение пробы на скрытую кровь в кале, е) Определение содержания трансаминаз,общего билирубина каждые 6-8 нед
Ответ: а,б,в,д,е

100. по поводу ревматизма длительно получает делагил.Какие вы предпримите меры по контролю безопасности терапии делагилом при его длительном применении: а)Общий анализ крови, б)Общий анализ мочи, в)ЭКГ, г)Исследование глазного дна, д)Исследование полей зрения, е)Рентгенологическое исследование органов грудной клетки, ж)Определение количества тромбоцитов, з) Исследование роговицы
Ответ: а,б,в,г,д,ж,з

101. 39 лет ревматоидный артрит, преимущественно суставная форма,2 степени активности.Какие варианты комбинированной терапии целесообразно назначить данному пациенту?
Ответ: Делагил по 0,25 г 3 раза в сутки, преднизолон по 15 мг/сут,кризанол в/м по 1 мл 5% раствора 1 раз в неделю

102. 63 лет страдает сахарным диабетом,принимает глибенкламид.Поступила в отделение с картиной острой правосторонней нижнедолевой пневмонии, подтвержденной рентгенологически.Был назначен хлорамфеникол,на который у б-ной отмечалась аллергическая реакция.Препарат отменили,выбран другой антибиотик-цефтриаксон.Однако при обследовании у б-ной выявлен низкий уровень клиренса креатинина (24 мл/мин),вследствие чего цефтриаксон был отменен.Каким препаратом следует продолжить лечение?
Ответ: Ципрофлоксацин

103. поступила в отделение с ж-ми на боли в правой молочной железе, повышение Т.до 39,5 С.Заболела 3 дня назад,на 10-й день после родов. При поступлении в отделение в верхненаружном квадранте правой молоч-
ной железы обнаружена гиперемия кожи,массивный инфильтрат с флуктуацией в центре.Диагноз:острый правосторонний мастит.Б-ная оперирована. Взят посев отделяемого раны.Определите антибиотик первого выбора
Ответ: Цефазолин

104. поступила в отделение с картиной острого правостороннего мастита.Заболела 3 дня назад,на 10-й день после родов. Б-ная оперирована.
Был назначен цефазолин.После 2-й инъекции препарата через 20 мин появились снижение АД, головокружение, тошнота, рвота, непроизвольное мочеиспускание, судорожный синдром. Какое осложнение развилось у пациентки?
Ответ: Анафилактическая реакция

105. Б-ная 21г.поступила в отделение с картиной острого правостороннего мастита.Заболела 3 дня назад, на 10-й день после родов.Б-ная опери рована.На цефазолин у б-ной отмечалась анафилактическая реакция,препарат был немедленно отменен.При посеве отделяемого раны выделены
стафилококк,образующий пенициллиназу,и гемофильная палочка.Выберите антибактериальный препарат с учетом бактериальной микрофлоры и особенностей фармакокинетики
Ответ: Левомицетин

106. страдает хроническим тонзиллитом и хроническим холециститом.В посеве отделяемого зева и в посеве желчи при обследовании выявлен золотистый стафилококк,образующий пенициллиназу.В анамнезе отмечена аллергия на оксациллин.Б-ной был назначен гентамицин.У б-ного клиренс креатинина составляет 50 мл/мин. Нужно ли корригировать режим дозирования препарата? Если да, то каким образом?
Ответ: Уменьшить кратность введения и снизить дозу

107. 50 лет обратилась с жалобами на общую слабость,жажду,частое мочеиспускание,зуд кожи и наружных половых органов.При осмотре выявлено ожирение (масса тела 96 кг при росте 168 см).Содержание глюкозы в крови 9,9 ммоль/л,в моче 1%, реакция на ацетон отрицательная. Какие гипогликемические препараты оптимальны в данном случае?
Ответ: Бигуаниды

108. 48 лет, поступил с жалобами на давящие боли, появляющиеся во время физической нагрузки, купирующиеся нитроглицерином. 3 года назад перенес инфаркт миокарда. В легких везикулярное дыхание. Тоны сердца приглушены, систолический шум на верхушке, частые экстрасистолы. ЧСС — 92 в мин. АД — 100/60 мм рт. ст. Печень не увеличена, отеков нет. ЭКГ — синусовая тахикардия, рубцовые изменения миокарда, частая желудочковая экстрасистолия. Назначен обзидан 160 мг/сутки, сустак-форте 19,2 мг/сутки, панангин, рибоксин.
Какой побочный эффект вероятен у больного при данной комбинации лекарственных средств?
Ответ: Гипотония, головокружение.

109. Больной М., 52 лет, поступил с жалобами на одышку, сердцебиение, боли в правом подреберье отеки на ногах. В течение 18 лет находится на диспансерном учете с диагнозом «ревматизм». Кожа бледная, акроцианоз, румянец щек. В базальных отделах легких-незвучные мелкопузырчатые хрипы. Границы относительной тупости средца расширены вверх и вправо. Тоны сердца приглушены, аритмичные, на верхушке систолический шум, акцент?? тона на легочной артерии. Пульс-96 в мин. ЧСС-140 в мин. АД — 130/85 мм рт. ст. Живот мягкий, печень на 3-4 см выступает из-под края реберной дуги. Отеки на ногах. Суточный диурез -650 мл. ЭКГ: отсутствует зубец Р, имеются волны «F-F», ритм неправильный. После в/в введения 10 мл 10% р-ра новокаинамида: восстановился синусовый ритм с ЧСС — 72 в мин, больному назначен новокаинамид внутрь по 0,5 г 4 раза в день, дигоксин 0,25 мг 1 табл.
3 раза в сутки, фуросемид 40 мг внутрь в течение 3-х дней. Через 5 дней у больного появилась тошнота, рвота, диарея, головокружение. ЭКГ: ритм синусовый,ЧСС-76 в мин, PQ -0,20 с, QRS-0,1с. Лечащий врач отменил дигоксин и фуросемид и назначил унитиол, препараты калия.
Оцените действия врача.
Ответ: Действия врача правильные, так как не только превышена средняя суточная доза дигоксина, но имеется и взаимодействие с новокаинамидом за связь с белком.

110. У больного Д., 53 лет, диагноз: ИБС, стабильная стенокардия III ФК, постинфарктный кардиосклероз, мерцательная аритмия,ХНК??Б ст. Принимал строфантин, дигоксин, фуросемид, панангин в средних терапевтических дозах. Неожиданно у больного повысилась температура до 38,4°С, появилась кашель, одышка, крепитация в легких справа. На рентгенограмме легких справа в нижней доле определяется участок инфильтрации. К лечению добавлены гентамицин, сульфокамфокаин,супрастин.
Возникновение каких побочных эффектов лечения наиболее вероятно у больного при такой комплексной терапии?
Ответ: При комбинации с фуросемидом наиболее вероятен нефротоксический эффект гентамицина.

111. Больной 28 лет поступил с жалобами на приступы сердцебиения, головную боль, озноб. Во время криза, который развивается 2-4 раза в году, повышается АД до 260/110 мм рт. ст., ЧСС-140 в мин., появляются бледность кожи, жгучие боли в области сердца, пульсация в голове, иногда повышение температуры тела до 38. После приступов полиурия. В межприступный период АД 120/80 мм рт. ст. При обьективном исследовании органической патологии со стороны внутренних органов не обнаружено. В анализах крови и мочи без патологии.
Укажите наиболее эффективный препарат (первый ряд) для купирования криза у больного:
Ответ: Фентоламин.

112. ревматоидным артритом на фоне курсового лечения метотрексатом возникло выраженное носовое кровотечение.Чем оно может быть обусловлено: а)Поражением сосудов носа вследствие основного патологического процесса, б)Повышением ПИ вследствие токсического гепатита,вызванного метотрексатом, в) Увеличением агрегации тромбоцитов под влиянием метотрексата,
г) Медикаментозно обусловленным снижением количества тро мбоцитов, д) Токсическим влиянием метотрексата на сосуды носа
Ответ: г,д

113. У больного К., 62 лет, АГ 1 степени. Последнее ухудшение состояния обусловлено психоэмоциональным стрессом. При осмотре: состояние относительно удовлетворительное, незначительная головная боль. АД-170/100 мм рт.ст. («рабочее» АД-120/70 мм рт.ст), ЧСС-90 в мин. Лечащий врач назначил анаприлин по 60мгсут, верапамил 160 мгсут.
Какие изменения Вы можете ожидать при назначении дополнительно к анаприлину верапамила?
Ответ: Усиление отрицательного дромотропного эффекта.

114. Больной С., 56 лет, по поводу стенокардии принимает нитросорбид (10 мг) 1т х 4р в день. Как изменится тактика проведения антиангинальной терапии, если у больного на фоне лечения нитратами возникнет мозговой инсульт?
Ответ: Отменить нитраты и назначить антиангинальный препарат из другой группы.

115. Больной 42 года с хроническим гломерулонефритом и артериальной гипертензией. При поступлении: АД 200/120 мм рт ст., пульс 75-80 уд в мин, отеки на лице, пояснице,голенях. Общий белок сыворотки крови 3.8 г%, в моче белок 16 г/л.. Укажите препараты, наиболее эффективные для гипотензивной терапии у данного больного:
Ответ: Эналаприл.

116. страдающему упорно рецидивирующим синдромом бронхиальной обструкции врач ввел п/к 1 мл адреналина.Какие проявления токсического
действия адреналина возможны в этой ситуации: а)Возбуждение ЦНС,б)Экстрасистолия, в)Токсическое поражение печени, г)Тахикардия, д)Блокада проведения импульса по проводящей системе сердца.
Ответ: а,б,г

117. 57 лет по поводу посинфарктного ардиосклероза,застойной сердечной недостаточности 2Б степени получает 40 мг фуросемида в/в и 300 мг
верошпирона внутрь.Какую диуретическую терапию Вы назначите б-ному в случае рефрактерности?
Ответ: Фуросемид 80 мг в/в и спиронолактон 300 мг внутрь

118. страдает неатопической бронхиальной астмой,сопровождающейся обильной бронхореей.Пульс 62 в мин.АД 140/80 мм.рт.ст. После назначения атропина сульфата у б-ной вначале отмечалось улучшение состояния-бронхорея резко уменьшилась,однако через 10 дней после начала лечения состояние вновь ухудшилось: появились лихорадка(37,8 С),одышка,кашель с трудно отделяемой мокротой, ЧСС 90 в минуту. Каковы причины подобных изменений состояния больной?
Ответ: Нарушение отхождения мокроты с ее последующим инфицированием

119. Женщина 52 лет страдает гипертонической болезнью?? ст. Принимает резерпин по 1 табл. (0.0001) 3 раза в день. АД нормализавалось через 1 неделю. Через 4 недели регулярного приема появились «голодные» боли в эпигастральной области, в ходе гастроскопии был диагностирован эрозивный дуоденит. Чем Вы обьясните его возникновение?
Ответ: Повышением тонуса n vagus на фоне резерпина и усилением желудочной секреции.

120. Больному 60 лет с ИБС, стабильной стенокардией IV ф.к. назначен кордарон 600 мг/сутки (в качестве антиангинального препарата).
Какие побочные эффекты могут наблюдаться у больного при длительном приеме кордарона?
Ответ: все перечисленное

121. При выборе режима дозирования лекарственных средств на основе Т?
определяют:
Ответ: кратность приема

122. Более точно характеризует скорость выведения лекарственных средств из организма:
Ответ: общий клиренс

123. Связь лекарственных средств с белками плазмы:
Ответ: определяет возможность развития побочных эффектов при сочетании лекарственных средств

124. Величина биодоступности важна для определения:
Ответ: пути введения лекарственных средств*

125. При длительном использовании сильных диуретиков может возникнуть:
Ответ: нарушение толерантности к глюкозе

126. Головокружение, отсутствие ощущения конечностей, затруднения при посадке и вставании без визуального контроля и другие симптомы токсического влияния встречаются у 75% пациентов, которые:
Ответ: получают стрептомицин

127. Передозировка симпатомиметиками вызывает:
Ответ: нарушения ритма

128. Побочные реакции, связанные с антибиотиком моксалактамом, включают в себе следующее:
Ответ: тромбоцитопению

129. Комбинированный прием индометацина и гентамицина наиболее часто вызывает:
Ответ: нарушение функции почек

130. Применение клавулановой кислоты в сочетании с амоксициллином позволяет:
Ответ: расширить спектр действия амоксициллина на штаммы бактерий, производящих бета-лактамазу

131. Больному, длительно получающему дифенин, планируется дополнительно назначить другой антиритмический препарат 1 класса, при назначении какого антиаритмика потребуется увеличение дозировки на 20-30% от стандартной?
Ответ: всех препаратов

132. Одновременный прием внутрь тетрациклина и препаратов Са2+ будет способствовать:
Ответ: снижению всасывания тетрациклина

133. Одновременно назначение хлорамфеникола и аценокумарола может привести:
Ответ: к снижению антибактериальной активности хлорамфеникола

134. При сердечной недостаточности:
Ответ: дофамин вызывает сужение сосудов коры почек в высоких дозах (более 10 мкг/кг/мин)

135. Для артериальной гипертензии характерно:
Ответ: увеличение концентрации натрия в сосудистой стенке

136. Апрессин (гидралазин):
Ответ: вызывает тахикардию

137. Бета-адреноблокаторы вызывают:
Ответ: снижение ЧСС

138. Справедливы такие утверждения об альфа-адреноблокаторах:
Ответ: все верно

139. Бета-1 — адреноблокаторы:
Ответ: избирательно действуют на бета1 -адренорецепторы, препараты безопасны при бронхиальной астме

140. Показаниями к применению бета-адреноблокаторов являются:
Ответ: нарушения сердечного ритма

141. Укажите правильные утверждения:
Ответ: строфантин в значительной степени разрушается в ЖКТ, в связи с чем прием его внутрь нерационален

142. Показания к назначению СГ:
Ответ: ХНК у больных ИБС, постинфарктным кардиосклерозом и постоянной формой мерцательной тахиаритмии

143. Фактор, повышающий риск развития интоксикации СГ:
Ответ: гипокалиемия

144. Состояние, повышающее риск развития интоксикации СГ:
Ответ: гипотиреоз

145. Для уменьшения риска развития толерантности к нитратам следует:
Ответ: делать перерывы между приемами препаратов

146. В случае развития толерантности к сустаку его можно заменить:
Ответ: корватоном

147. Головную боль может вызвать прием:
Ответ: верны ответы А,Б,В

148. Сходным по механизму действия с нитроглицерином является:
Ответ: молсидомин

149. При передозировке каких препаратов может возникнуть ортостатическая гипотония?
Ответ: нитратов

150. Назовите группу антиаритмиков, увеличивающих продолжительность потенциала действия:
Ответ: блокаторы калиевых каналов

151. Какой из перечисленных препаратов оказывает наиболее выраженное отрицательное инотропное действие?
Ответ: дизопирамид

152. Какие из некардиальных побочных эффектов характерны для большинства препаратов 1С класса?
Ответ: нарушение зрения

153. При терапии дизопирамидом может обостриться следующее заболевание:
Ответ: доброкачественная гиперплазия простаты с нарушением мочеиспускания

154. В каких случаях дозировка лидокаина должна быть изменена по сравнению со стандартной?
Ответ: у пациентов с печеночной недостаточностью

155. Укажите методы контроля за эффективностью применения диуретиков при отечном синдроме:
Ответ: все верно

156. Укажите методы контроля за безопасностью применения диуретиков при отечном синдроме:
Ответ: все верно

157. Укажите эффективный и безопасный способ пополнения запасов калия в организме:
Ответ: назначение панангина внутрь по 2 таблетки 3 раза в день

158. Укажите факторы риска возникновения побочных эффектов «петлевых» диуретиков:
Ответ: суточный диурез более 3 л после введения диуретика

159. Укажите начало действия спиронолактона:
Ответ: 4-5 дней

160. Для ощелачивания мочи используют следующие методы, кроме:
Ответ: цитрат калия 3 мг каждые 6 часов

161. Реакция мочи может быть кислой при использовании следующих препаратов, кроме:
Ответ: метионин

162. Отметьте ошибочные положения:
Ответ: нет ошибочных положенией

163. Распределите указанные препараты по степени кумуляции:
Ответ: неодикумарин

164. Отберите утвеждения, которые являются полностью правильными для препарата стрептокиназы:
Ответ: все верно

165. Отберите фактор, вызывающий тромбоз или способствующий тромообразованию:
Ответ: все верно

166. Какой из следующих лекарственных препаратов может уменьшить эффект антикоагулянтов непрямого действия?
Ответ: рифампицин

167. Какие побочные эффекты могут встречаться при применении гепарина?
Ответ: все перечисленное

168. Больному с бронхиальной астмой, длительно получавшему пролонгированные теофиллины, в связи с развитием инфекции мочевыводящих путей назначают ципрофлоксации. В этом случае необходимо:
Ответ: уменьшить дозу теофиллинов на 30%

169. У ребенка, длительно получающего карбамазепин в связи с наличием эпилепсии, развивается бронхообструктивный синдром с дыхательной недостаточностью 2-й ст. При назначении аминофиллина такому пациенту:
Ответ: доза аминофиллина должна быть увеличена в 1.5 раза

170. При назначении теофиллина курильщику:
Ответ: доза должна быть увеличена

171. Укажите препарат, снижающий элиминацию теофиллина при одновременном назначении:
Ответ: циметидин

172. У больному бронхиальной астмой, длительно получавшего теотард, на фоне гриппозной инфекции и лихорадки появилась тошнота, рвота, головные боли, бессонница. Менингиальные симптомы отрицательны. Терапевтическая тактика в данном случае:
Ответ: отменить теотард или снизить его дозу на 50%

173. Побочные эффекты теофиллина могут включать следующие явления, кроме:
Ответ: развития отечного синдрома

174. Специфическим побочным эффектом, возникающим при применении теофиллина у детей 1-го года жизни, является:
Ответ: мелена

175. Укажите ингаляционный глюкокортикостероидный препарат, обладающий наименьшей биодоступностью:
Ответ: флутиказона пропионат

176. Обозначьте ингаляционный глюкокортикостероидный препарат, обладающий наименьшим сродством к глюкокортикостероидным рецепторам легких человека:
Ответ: флутиказона пропионат

177. Укажите препарат, обладающий наибольшей степенью безопасности (по индексу безопасности:)
Ответ: преднизолон

178. Какой из глюкокортикостероидных препаратов в наибольшей мере способствует развитию миопатии?
Ответ: триамцинолон

179. Замедление выделения из организма натрия и воды, усиление выведения калия (минералокортикоидный эффект) в большей степени характерна:
Ответ: гидрокортизону

180. Минералокортикоидная активность отсутствует у:
Ответ: дексаметазона

181. Выберите правильный ответ. Глюкокортикоиды:
Ответ: являются контринсулярными гормонами

182. При проведении пульс-терапии более предпочтителен:
Ответ: метилпреднизолон

183. При назначении на длительный срок предпочтительнее использовать:
Ответ: преднизолон

184. Какой блокатор Н1-гистаминорецепторов противопоказан при анафилактическом шоке?
Ответ: дифенгидрамин (димедрол)

185. Выберите оптимальный блокатор Н1-гистаминорецепторов для лечения аллергического ринита:
Ответ: азеластин (аллергодил)

186. Обозначьте лекарственное средство из группы стабилизаторов мембран тучных клеток в лекарственной форме в виде порошка для ингаляций:
Ответ: кромоглициевая кислота (бикромат)

187. К иммуностимулятору микробного происхождения относится:
Ответ: рибомунил

188. Основное показание для назначения рибомунила заключается в:
Ответ: профилактике рецидивирующих инфекций верхних дыхательных путей

189. Хорошо проникают через гематоэнцефалический барьер следующие антибактериальные препараты:
Ответ: цефалоспорины III генерации

190. Новое поколение макролидных антибиотиков имеет следующие преимущества, кроме:
Ответ: почечный путь экскреции

191. Фторхинолоны отличаются от хинолонов следующими свойствами, кроме:
Ответ: бактериостатическим действием

192. Отметьте, какие утверждения в отношении цефалоспоринов правильны:
Ответ: все верно

193. Последствия приема антибиотиков включают:
Ответ: все верно

194. Укажите препарат выбора при инфекции мочевых путей, вызванной синегнойной палочкой:
Ответ: цефтазидим

195. Какие препараты показаны для лечения хламидийной инфекции мочеполового тракта:
Ответ: ровамицин

196. Укажите препарат с наименее выгодными фармакокинетическими характеристиками:
Ответ: кетоконазол

197. Укажите антимикотический препарат, не подвергающийся метаболизму в печени:
Ответ: флуконазол

198. Укажите антимикотический препарат (из группы аллиламинов), применяемый в первую очередь для лечения дерматомикозов:
Ответ: тербинафин

199. Укажите клиническое состояние, являющееся показанием для проведения монотерапии НПВС:
Ответ: внесуставные ревматические заболевания (миозит, тендовагинит, синовит)

200. Для ацетилсалициловой кислоты характерно:
Ответ: при оральном приеме всасывается в основном из верхнего отдела тонкого кишечника

201. В сравнении с индометацином у ацетилсалициловой кислоты более выражено:
Ответ: антиагрегантное действие на тромбоциты

202. На скорость выведения ацетилсалициловой кислоты и ее метаболитов влияют:
Ответ: уровень рН мочи

203. Желудочно-кишечные осложнения при применении ацетилсалициловой кислоты связаны с:
Ответ: все перечисленное

204. Для фенилбутазона характерно:
Ответ: все верно

205. При взаимодействии индометацина с другими препаратами:
Ответ: уменьшается диуретическая активность фуросемида

206. Какие побочные реакции НПВС коррегирует комплексный препарат артротек (диклофенак натрия + мизопростол)
Ответ: НПВС-гастропатии

207. Какие характеристики парацетамола выдвинули этот препарат на первое место в ряду анальгетиков-антипиретиков?
Ответ: более раннее наступление анальгетического и жаропонижающего эффекта

208. Выберите препарат, селективно ингибирующий циклооксигеназу2:
Ответ: мелоксикам

209. Наилучший анальгезирующий эффект фентанила наблюдается в комбинации с:
Ответ: дроперидолом

210. Назовите противовоспалительное средство пролонгированного действия:
Ответ: пироксикам

211. У Б-ой 52л.развилась картина гипертонического криза 2 типа с ЧСС 62 в мин.АД 200/140 мм.рт.ст.В легких большое кол-во влажных мелкопузырчатых хрипов.С какого препарата необходимо начинать купирование криза:
Ответ: Фуросемид

212. в течение 6 лет страдает сахарным диабетом 1 типа, получает инсулин по 54 ЕД/сут,что поддерживает уровень гликемии в пределах 7,0 ммоль/л.В последнее время в связи с повышением АД до 16090 мм.рт.ст. лечащий врач назначил гипотиазид в суточной дозе 75 мг в комбинации с эналаприлом в дозе 5 мг.Через 10 дней у больного уровень сахара в крови 10,5 ммольл, отмечается ухудшение самочувствия. Что является ведущей причиной изменения уровня сахара в крови?
Ответ: Комбинация эналаприла с гипотиазидом

213. развилась судорожная форма гипертонического криза, состояние тяжелое, цифры АД 200120 мм.рт.ст., ЧСС 120 в мин. С назначения какого препарата необходимо начинать терапию?
Ответ: Диазепама

214. на фоне применения антибиотика цефтриаксона в течение 10 дней развилась картина псевдомембранозного колита. Какой первый шаг алгоритма врачебной помощи?
Ответ: отмена цефтриаксона, назначение ванкомицина или метронидазола

215. по поводу обострения язвенной болезни желудка в комплексе терапии назначен кларитромицин. Назовите основные отличительные признаки препарата от эритромицина.
Ответ: все верно

216. после перенесенной операции на брюшной полости на 4-е сутки развилась левосторонняя нижнедолевая пневмония. Результаты экспресс-анализа показали наличие MRSA, пенициллино- и аминогликозидорезистентных штаммов энтерококков. Препараты выбора:
Ответ: Ванкомицин

217. находится в отделении интенсивной терапии по поводу синегнойной инфекции. Выберите препараты 1 ряда для лечения?
Ответ: Цефтазидим + аминогликозиды

218. 40 лет без сопутствующих заболеваний по поводу внебольничной пневмонии в амбулаторных условиях был назначен спирамицин внутрь по 3 млн. МЕ 2 рс, на 2-е сутки лечения отмечались интенсивные гастралгии, тошнота, однократная рвота. Выберите альтернативный препарат.
Ответ: доксициклин

219. с хроническим обструктивным бронхитом выявлена пневмония средней тяжести, в амбулаторных условиях назначен амоксиклав внутрь по 625 мг 3 рс. На 2-е сутки у больного развилась крапивница, бронхоспазм. Назовите альтернативный препарат для лечения пневмонии.
Ответ: моксифлоксацин внутрь

220. У ВИЧ- инфицированного б.44 лет диагностирована пневмоцистная пневмония. Назовите препарат для лечения?
Ответ: ко-тримоксазол вв 20 мгкгс 4 рс в течение 21 сут

221. У б-ной 28л. отмечаются ежедневные симптомы бронхиальной астмы, частые обострения, частые ночные симптомы, диагностирована тяжелая персистирующая бронхиальная астма. Назовите препараты базисной терапии.
Ответ: ингаляционные глюкокортикоиды (больше 1000 мкг беклометазона дипропионата) + ингаляционные бета-2-агонисты длительного действия

222. К врачу обратилась беременная женщина (срок беременности 6-7 нед.) с клиническими признаками острой пневмонии. Какие группы антибактериальных препаратов разрешены к применению у беременных?
Ответ: цефалоспорины

223. 57 лет по поводу артериальной гипертензии умеренной степени получает в монорежиме ингибитор АПФ — эналаприл. У больного через 2 года приема препарата отмечается недостаточность эффекта. Какой наиболее приемлемый вариант оптимизации терапии?
Ответ: добавление к препарату диуретика (гипотиазида или индапамида)

224. получает антибактериальный препарат по поводу инфекционного процесса. При внутривенной инфузии препарата отмечается реакция в виде выраженного покраснения кожных покровов верхней половины туловища, лица, шеи, симптомы значительно уменьшаются при снижении скорости инфузии. На какой препарат отмечается такая реакция?
Ответ: Ванкомицин

225. У беременной женщины отмечается активация ревматического процесса. Какой препарат из группы антикоагулянтов можно назначить беременной?
Ответ: Гепарин

226. поступил с ж-ми на повышение АД до цифр 15090 мм.рт.ст. на фоне психоэмоционального перенапряжения, сердцебиение, тревогу, нарушение сна. Год назад выявлен сахарный диабет 2 типа, получает манинил. Назовите препарат выбора для лечения АГ.
Ответ: Атенолол

227. в связи с симптомами стенокардии и нарушения ритма назначены препараты: анаприлин 200 мгс и верапамил 240 мгс в течение длительного времени. Какие возможны побочные реакции?
Ответ: Развитие а-в блокад, брадикардия

228. Женщина 34 лет принимает эстрогенсодержащие контрацептивные препараты. Лечащий врач назначил доксициклин в дозе 200 мгс в течение 2 недель. Какое вероятное взаимодействие ожидается?
Ответ: Снижается эффект контрацепции

229. Анестезиолог при введении кетамина с профилактической целью назначил больному диазепам. Профилактика какого состояния проводится таким образом?
Ответ: Постнаркозных галлюцинаций

230. 46 лет поступил с картиной острого деструктивного аппендицита. Препарат выбора для антибиотикопрофилактики?
Ответ: Цефазолин

231. К врачу обратился больной с жалобами на кашель, повышение температуры до 39 С, боли в грудной клетке. Диагностирована правостронняя бронхопневмония. Назначен препарат в течение 3 суток, обладающий постантибиотическим эффектом. Назовите препарат выбора.
Ответ: Азитромицин

232. по поводу острого правостороннего пиелонефрита назначен цефазолин по 2 гс в течение 10 дней. Назовите наиболее типичную ошибку при выборе данного антибиотика
Ответ: Недостаточно высокая активность в отношении грамотрицательной флоры

233. одновременно получает в течение 14 дней фторхинолоновый антибиотик — офлоксацин по поводу инфекции мочевыводящих путей и диклофенак натрия в связи с суставным синдромом. Какое вероятное взаимодействие ожидается?
Ответ: повышение риска возбуждения ЦНС и развития судорог

234. после переохлаждения возникли озноб,повышение Т.тела до 38,6С, кашель с отделением слизисто-гнойной мокроты,боли в левой половине грудной клетки.Клинически и рентгенологически установлен диагноз левосторонней нижнедолевой пневмонии.Б-ной назначено лечение:цефазолин по 1 г 2р.в сутки вм,гемодез 400 мл в/в капельно, отхаркивающая микстура по 1ст.л.6р.в сутки. На 3-е сутки отмечалась реакция в виде крапивницы, кожного зуда. Выберите антибактериальный препарат для замены?
Ответ: Спирамицин

235. При проведении наркоза анестезиолог с целью антибиотикопрофилактики назначил антибактериальный препарат. У больного развилась остановка дыхания. Какой препарат использовался?
Ответ: Гентамицин

236. в течение 10 лет страдает деформирующим остеоартрозом нижних конечностей с выраженными синовитами. В анамнезе отмечает лекарственную аллергию на бутадион. В отделении б-ной назначены реопирин по 5 мл в/м 1 раз в сутки.Через день у б-ной появились зудящие эритематозные высыпания на коже туловища.Какова наиболее вероятная причина ухудшения состояния?
Ответ: Лекарственная аллергическая реакция

237. по поводу нарушения ритма сердца назначен новокаинамид, одновременно в связи с сезонным аллергическим ринитом назначен цетиризин. Какое вероятное взаимодействие ожидается?
Ответ: тяжелые формы аритмии (типа пируэта)

238. Пациенту с СКВ был назначен метотрексат. Через какое время ожидается стабильное проявление лечебного эффекта?
Ответ: бірнеше айдан кейін

239. с гипертоническим кризом был назначен препарат фозиноприл. Несмотря на прием адекватной дозы препарата не отмечено снижение уровня АД в ближайшие минуты и часы. Назовите причину.
Ответ: При кризах не используются депо-препараты

240. с целью плановой терапии АГ был назначен апрессин в течение длительного времени. У больного через месяц применения препарата стали отмечаться сердцебиение, стенокардитические боли, снижение эффекта от лечения. Назовите основную причину развившихся явлений
Ответ: Апрессин не назначается для плановой терапии АГ

241. 42 лет поступил в отделение интенсивной терапии по поводу тяжелого гипертонического криза. В течение 5 суток внутривенно инфузионно вводился нитропруссид натрия. На 6-е сутки у больного развилась картина интоксикации в виде неукротимой рвоты, снижения функции сердечно-сосудистой, дыхательной и выделительной систем. Назовите основную причину развившегося состояния.
Ответ: Передозировка препарата (накопление тиоцианатов в крови)

242. 54 лет страдает сахарным диабетом 1 типа,принимает пролонгированный инсулин.Поступила в отделение с картиной острой правосторонней нижнедолевой пневмонии, подтвержденной рентгенологически.Был назначен цефтриаксон,на который у б-ной отмечалась аллергическая реакция.Препарат отменили,выбран другой антибиотик- ципрофлоксацин в сочетании с амикацином.Однако при обследовании у б-ной выявлен низкий уровень клиренса креатинина (30 мл/мин),вследствие чего амикацин был отменен.Каким препаратом следует продолжить лечение?
Ответ: Спирамицин

243. поступила в отделение с ж-ми на боли в низу живота, повышение Т.до 39,5 С.Заболела 2 дня назад,на 6-й день после родов. При гинекологическом осмотре — картина острого послеродового эндометрита. Результаты бакпосева: золотистый стафилококк, образующий пенициллиназу, протей. Определите антибиотик первого выбора
Ответ: Цефепим

244. 25 л.поступила в отделение с картиной острого правостороннего пиелонефрита.Заболела 3 дня назад после переохлаждения. Был назначен цефазолин.После 2-й инъекции препарата через 10 мин появились снижение АД, головокружение, тошнота, рвота, непроизвольное мочеиспускание, судорожный синдром. Какое осложнение развилось у пациентки?
Ответ: Анафилактическая реакция

245. Б-ная 28 л.поступила в отделение с картиной острого правостороннего мастита.Заболела на 12-й день после родов.Б-ная оперирована.На цефазолин у б-ной отмечалась анафилактическая реакция,препарат был немедленно отменен.При посеве отделяемого раны выделены стафилококк,образующий пенициллиназу,и кандиды.Выберите антибактериальный препарат с учетом бактериальной микрофлоры и особенностей фармакокинетики
Ответ: Оксациллин + флуконазол

246. страдает хроническим холециститом.В посеве желчи при обследовании выявлены золотистый стафилококк и кишечная палочка. В анамнезе отмечена аллергия на оксациллин. Препараты выбора.
Ответ: Цефтриаксон

247. 58 лет обратилась с жалобами на общую слабость,жажду,частое мочеиспускание,зуд кожи и наружных половых органов.При осмотре: масса тела 56 кг при росте 168 см).Содержание глюкозы в крови 12,3 ммоль/л,в моче 1,5%, реакция на ацетон отрицательная. Какие гипогликемические препараты оптимальны в данном случае?
Ответ: Препараты сульфонилмочевины

248. 53 лет, поступил с жалобами на сердцебиение, перебои, временами одышку. Эти явления стали беспокоить после перенесенного инфаркта миокарда 2 года назад. Прием новокаинамида в течение 3 месяцев принес значительное облегчение. Однако в последнее время самочувствие ухудшилось. Дальнейшая тактика ведения больного.
Ответ: полное обследование и выбор препарата

249. 33 лет по поводу послеоперационного гнойного перитонита получает комбинацию препаратов: цефтриаксон + амикацин + метронидазол. В анамнезе — желчнокаменная болезнь. Назовите препарат, который не должен назначаться больному.
Ответ: Цефтриаксон

250. 45 лет по поводу кандидозной пневмонии был назначен флуконазол вв в течение 3 дней, затем внутрь. На 4-е сутки лечения отмечена реакция в виде сильных головных болей, тошноты. Решено было заменить препарат на кетоконазол. Оцените адекватность тактики.
Ответ: Кетоконазол не является адекватной заменой в силу невыгодных фармакокинетических характеристик

251. 42 лет, поступил с жалобами на выраженное сердцебиение, повышение цифр АД до 240140 мм.рт.ст. При обследовании выявлено значительное повышение в крови уровня катехоламинов. Назовите препарат выбора для купирования криза.
Ответ: фентоламин

252. 50 лет, поступил с картиной острого левостороннего пиелонефрита. Были назначены цефазолин + гентамицин в среднетерапевтических дозах. У больного при обследовании клиренс креатинина — 50 млмин. Какие вероятные последствия терапии?
Ответ: Риск нефротоксичности

253. 48 лет с пароксизмальной над- и желудочковой тахикардией был назначен кордарон вв в первые сутки, затем внутрь. При обследовании выявлено нарушение функции щитовидной железы, а-в блокада 2-3 степени. Дальнейшая тактика.
Ответ: Отмена препарата, назначение новокаинамида

254. 50 лет находится в отделении кардиореанимации по поводу острого инфаркта миокарда, получает комплекс терапии. Какие основные параметры мониторинга при назначении прямых антикоагулянтов?
Ответ: АЧТВ, время свертывания крови, моча на эритроциты

255. В отделении находится больной после операции по поводу аппендицита. Лечащий врач назначил гентамицин по 80 мг 3 рс вм. Пациент 2 года назад перенес острый гломерулонефрит, в настоящее время клиренс креатинина составляет 50 млмин. Какая коррекция требуется?
Ответ: замена на препарат, не обладающий нефротоксичностью

256. 56 лет, получает дигоксин по 0,25 гс в течение последнего года. В настоящее время стало отмечаться повышение цифр АД до 180110 мм.рт.ст. Лечащим врачом назначен лизиноприл в дозе 10 мгс. Через 3 месяца у пациента диагностирована дигиталисная интоксикация. Дальнейшая тактика.
Ответ: назначение гипотензивного препарата другого ряда

257. 53 лет диагностирован кандидозный и аспергиллезный менингит. Препараты выбора.
Ответ: Амфотерицин В

258. 58 лет, получающего длительное время глюкокортикоиды в комплекс терапии был включен синтетический антибиотик широкого спектра в связи с инфекцией репродуктивной сферы. На 14 сутки совместного применения отмечено тяжелое осложнение в виде разрыва ахиллова сухожилия. Назовите антибиотик, который в сочетании с ГКС вызвал данное осложнение.
Ответ: Левофлоксацин

259. в отделение поступил Б.43 лет с картиной острой хламидийной пневмонии. Назовите препараты выбора.
Ответ: Ровамицин

260. У больного 24 лет диагностирована неосложненная форма гонореи. Назовите препарат выбора.
Ответ: Цефтриаксон

261. Выберите правильное утверждение: а)биодоступность-количество ЛС,поступающее в системный кровоток,выраженное в процентах от введенной дозы,б)биодоступность определяется величиной адсорбции ЛС в ЖКТ и выра-
женностью эффекта первого прохождения через печень.в)биодоступность определяют по формуле: F = AUC (в/м или внутрь)/AUC (в/в).г)биодоступность ЛС при внутримышечном введении определяется степенью его всасывания и биотрансформации в организме.
Ответ: а, б, в

262. страдает неатопической бронхиальной астмой,сопровождающейся обильной бронхореей.Пульс 62 в мин.АД 140/80 мм.рт.ст.Какие препараты более предпочтительны?
Ответ: Атровент

263. Механизмы всасывания ЛС в кишечнике:
а) пассивная диффузия, б) фильтрация, в) активный транспорт,г) облегченный транспорт, д) пиноцитоз:
Ответ: а, д

264. поступил с ж-ми на изжогу,боли в эпигастральной области натощак, купируемые приемом натрия гидрокарбоната.При ФЭГДС выявлена язва (0,5см в диаметре) в ампуле 12 п.к.РН-метрия желудочного сока: кислотообра-
зующая функция средней интенсивности с низкими щелочными резервами,холинергический тип рецепции. Диагноз: язвенная болезнь 12 п.к.в стадии обострения. Выберите наиболее эффективное и безопасное ЛС и определите режим его дозирования:
Ответ: Пирензепин до еды по 0,05 г 3 раза в сутки в течение 2дней, затем по 0,05 г 2 раза в сутки

265. выявлена дискенезия желчного пузыря по гипертоническому типу. Выберите оптимальный вариант лечения.
Ответ: Но-шпа по 1-2 табл.3 раза в сутки, отвар бессмертника по 1/2 стакана за 30 мин до еды

266. страдает хрон.холецистопанкреатитом в течение 5 лет.За последнюю неделю после нарушения диеты отмечает усиление болей в правом по дреберье,тошноту,горечь во рту.Выберите наиболее эффективные желчегонные средства, обладающие одновременно противомикробной активностью:
а) Аллохол, б) Холензим, в) Никодин, г) Отвар пижмы,
д) Ксилит
Ответ: а,в

267. с суицидальной целью выпила 20 таблеток феназепама.Через 2 ч после приема препарата доставлена в стационар. Б-я в сознании,но резко заторможена.Проведено промывание желудка.Выберите наиболее оптимальные слабительные средства: а) Глауберова соль, б) Сульфат магния, в) Экстракт коры крушины, г) Бисакодил,
д) Касторовое масло, е) Морская капуста, ж) Вазелиновое масло
Ответ: а,б,д

268. 46 лет поступил в отделение кардиореанимации с острым трансмуральным инфарктом миокарда, возникшим около 5 ч назад.Назначения:анаприлин 20 мг 4 раза в сутки внутрь, гепарин в/в капельно по 10 000 ЕД каждые 4 часа.При этом удалось достигнуть увеличения времени свертывания крови до 18-23 минуты. На 4-й день у б-ного выявлена микрогематурия (22 эритроцита в поле зрения). Какова ваша тактика?
Ответ: Снизить дозу гепарина вплоть до времени свертывания крови не менее 10-12 мин

Волгоград 2011 год


ТЕСТЫ ПО КЛИНИЧЕСКОЙ ФАРМАКОЛОГИИ ДЛЯ СТУДЕНТОВ V КУРСА ПЕДИАТРИЧЕСКОГО ФАКУЛЬТЕТА

001. Равновесная концентрация препарата (Сss) это:

а) Концентрация препарата, при которой развивается максимальный терапевтический эффект.

б) Концентрация препарата, при которой развивается минимальный терапевтический эффект.

в) Установившаяся концентрация препарата при его повторном введении, при которой количество вводимого препарата равно количеству выводимого препарата.

г) Установившаяся концентрация препарата при его повторном введении, при которой появляются первые побочные эффекты препарата.

002. Элиминация вещества это:

а) Суммарный результат экскреции и метаболизма вещества.

б) Результат метаболизма вещества.

в) Результат экскреции вещества печенью.

г) Результат экскреции вещества почками.

003. Клиренс препарата это:

а) Процент снижения концентрации вещества в крови в единицу времени.

б) Количество вещества в процентах от принятой дозы, элиминируемое в течение суток.

в) Объём крови, который освобождается от лекарственного средства в единицу времени.

г) Скорость снижения концентрации вещества в единицу времени.

004. Абсорбция лекарственных средств не зависит от:

а) Активности микросомальных ферментов печени.

б) Пути введения ЛС.

в) Растворимости ЛС в воде и липидах.

г) Интенсивности кровотока в месте введения ЛС.

005. Величина фармакологического эффекта зависит от:

а) Концентрации лекарственного средства в биофазе.

б) Времени введения препарата.

в) Скорости введения препарата.

г) Концентрации лекарственного средства в жировой ткани.

006. Механизм действия ингибиторов АПФ реализуется через:

а) Влияние на активность ферментов.

в) Физико-химическое влияние на мембраны клеток.

007. Равновесная концентрация препарата (Сss) устанавливается через:

а) 1-2 периода полувыведения препарата.

б) 4-6 периодов полувыведения препарата.

в) 2-3 периода полувыведения препарата.

г) 7-9 периодов полувыведения препарата.

008. Тахифилаксия это:

а) Быстрое развитие толерантности после нескольких повторных приёмов ЛС.

б) Накопление ЛС в организме.

в) Быстрое выведение ЛС из организма.

г) Угнетение активности ферментных систем печени.

009. Укажите индуктор микросомального окисления:

а) Фенобарбитал.

б) Циметидин.

в) Амиодарон.

г) Ловастатин.

010. Укажите ингибиторы микросомального окисления:

а) Противосудорожные препараты.

б) Макролиды.

в) Противотуберкулёзные препараты (изониазид, рифампицин).

г) Алкоголь.

011. Повышение pH мочи ускоряет выведение:

а) Слабых кислот.

б) Слабых оснований.

012. Укажите препарат, для которого значимо возможное вытеснение из связи с альбуминами:

а) Варфарин.

б) Атенолол.

в) Гентамицин.

г) Будесонид.

013. Укажите ингибитор микросомального окисления в печени:

а) Грейпфрутовый сок.

б) Алкоголь.

в) Пища, приготовленная на углях.

г) Сигаретные смолы.

014. Терапевтическим лекарственным мониторингом называется:

а) Контроль за скоростью выведения препарата.

б) Контроль за правильностью введения препарата.

в) Контроль за равновесной концентрацией препарата.

г) Контроль за побочными эффектами препарата.

015. Метод, позволяющий случайно включать пациентов в оду из групп при проведении клинических исследований, называется:

а) метаанализ.

б) рандомизация.

в) стратификация.

г) сортировка.

016. К фазам клинических исследований не относится:

а) Исследования на животных.

б) Исследования на здоровых добровольцах.

в) Исследования на ограниченном числе больных.

г) Исследования на большом количестве больных

017. I фаза клинических исследований включает:

а) Исследования на здоровых добровольцах.

б) Исследования на ограниченном числе больных.

в) Исследования на большом количестве больных.

г) Исследования на животных.

018. Тератогенный эффект ЛС:

а) Формирование пороков развития у плода.

б) Гибель зародыша до установления беременности.

в) Развитие внутриутробной гипотрофии плода, функциональная незрелость органов.

019. Препараты, для которых обнаружено наличие риска для человека, однако польза от их применения во время беременности превышает возможный риск, относятся к категории:

020. У новорожденных объем распределения:

а) Для водорастворимых препаратов повышен, для жирорастворимых - понижен.

б) Для водорастворимых препаратов понижен, для жирорастворимых - повышен

в) Для водорастворимых препаратов понижен, для жирорастворимых - понижен

г) Для водорастворимых препаратов повышен, для жирорастворимых - повышен

021. Увеличение свободной фракции ЛС в крови новорожденного связано:

а) С меньшим количеством белков в плазме крови.

б) С низкой скоростью клубочковой фильтрации.

в) С небольшой массовой долей подкожно-жировой клетчатки с высоким процентом воды.

г) С небольшой мышечной массой.

022. Дженерический препарат фармакокинетически эквивалентен оригинальному если эквивалентны:

а) Максимальная концентрация, время достижения максимальной концентрации, площадь под кривой концентрация-время;

б) Максимальная концентрация, время достижения максимальной концентрации, период полувыведения.

в) Биодоступность, период полувыведения.

023. К методам фармакоэкономического анализа не относятся:

а) Метод затраты-эффективность.

б) Метод затраты-прибыль.

в) Метод затраты-полезность.

г) Метод затраты-выгода

024. Метод фармакоэкономического анализа, при котором все результаты представлены в денежном эквиваленте называется:

а) Минимизация затрат.

б) Затраты-полезность.

в) Затраты-выгода.

г) Затраты-эффективность.

025. Хронофармакология изучает:

а) Фармакокинетическое и фармакодинамическое взаимодействие ЛС с организмом в зависимости от биоритмов.

б) Активность ферментов печени в зависимости от биоритмов.

в) Влияние различных ЛС на биоритмы.

001. Целевым уровнем АД у детей с артериальной гипертензией является:

а) Менее 90-го перцентиля кривой распределения АД.

б) Менее 95-го перцентиля кривой распределения АД.

в) Менее 85-го перцентиля кривой распределения АД.

002. При вторичной артериальной гипертензии у детей медикаментозная терапия назначается:

а) Через 6 месяцев после неэффективности немедикаментозной терапии;

б) Через 12 месяцев после неэффективности немедикаментозной терапии;

в) Сразу после постановки диагноза;

г) Не назначается.

003. Суррогатным критерием эффективности терапии артериальной гипертензии является:

а) Снижение систолического давления на 10 мм.рт.ст.;

б) Достижение целевого уровня АД;

в) Снижение систолического и диастолического давления на10 мм.рт.ст.;

004. Нерациональной комбинацией гипотензивных препаратов является:

а) Диуретик + бета-блокатор.

б) Антагонист кальция + диуретик.

в) Антагонист кальция + бета-блокатор.

г) Бета– блокатор + ИАПФ.

005. Нефропротективным действием не обладают:

а) Бета-адреноблокаторы.

в) Антагонисты кальция.

г) Блокаторы рецепторов к ангиотензину II

006. При артериальной гипертензии с сопутствующей тахиаритмией препаратами выбора являются:

а) Бета-адреноблокаторы.

б) Диуретики.

г) Препараты центрального действия.

007. Препарат выбора для лечения артериальной гипертензии у беременных:

а) Клонидин.

б) Метилдофа.

в) Гуанфацин.

г) Моксонидин.

008. Выберете неселективные бета-блокаторы с вазодилатирующими свойствами:

а) Карведилол, лабеталол

б) Талинолол, ацебутолол

в) Атенолол, метопролол,

г) Пропранолол, надолол, соталол

009. Изменение ЧСС под действием бета-блокаторов:

а) Уменьшают ЧСС в покое (все) и при физической нагрузке (без ВСА).

б) Уменьшают ЧСС в покое (без ВСА) и при физической нагрузке (все).

в) Уменьшают ЧСС в покое (с ВСА) и при физической нагрузке (все).

г) Уменьшают ЧСС в покое (все) и при физической нагрузке (с ВСА).

010. Назначение больным с лабильным сахарным диабетом I типа опасно в связи с тем, что эти препараты

а) Маскируют симптомы гипогликемии.

б) Уменьшают высвобождение инсулина бета-клетками поджелудочной железы..

в) Тормозят мобилизацию глюкозы в печени в случае гипогликемии.

г) Снижают толерантность больных к глюкозе.

011. Среди всех антагонистов кальция единственным препаратом, блокирующим кальциевые каналы Т-типа, является:

а) Дилтиазем

б) Нифедипин

в) Мибефрадил.

г) Верапамил.

012. Дигидропиридины блокируют кальциевые каналы с преимущественным расположением:

а) В AV – узле.

б) В артериях.

в) И в артериях, и в AV – узле.

013. Механизм действия антагонистов кальция реализуется через:

а) Физико-химическое влияние на мембраны клеток.

б) Влияние на специфические рецепторы.

в) Влияние на активность ферментов.

г) Прямое химическое взаимодействие ЛС.

014. Антагонисты кальция дигидропиридинового ряда противопоказаны при:

а) Аортальном стенозе.

б) Митральном стенозе.

в) Аортальной недостаточности.

015. Изменения водно-электролитного баланса на фоне терапии ИАПФ:

а) Выведение калия, задержка натрия и воды.

б) Выведение калия, натрия и воды.

в) Выведение натрия и воды, задержка калия.

г) Выведение воды, задержка калия и натрия.

016. Выберите ИАПФ, который имеет два пути выведения:

а) Трандолаприл.

б) Квинаприл.

в) Фозиноприл.

г) Периндоприл.

017. Выберите группу препаратов, предпочтительную в лечении артериальной гипертензии на фоне выраженной почечной недостаточности:

б) Блокаторы рецепторов к ангиотензину II

в) Бета-блокаторы

г) Антагонисты кальция

018. Терапия ИАПФ сопровождается:

а) Повышением чувствительности периферических тканей к инсулину.

б) Снижением чувствительности периферических тканей к инсулину.

в) Ухудшением липидного профиля крови.

019. Преимуществом блокаторов АТ1 рецепторов перед ИАПФ является:

а) Возможно назначение при развитии ангионевротического отека на фоне лечения ИАПФ.

б) Возможно назначение при исходной гиперкалиемии.

в) Возможно назначение при стенозе почечных артерий.

020. Диуретики, используемые для длительной терапии артериальной гипертензии:

а) Петлевые.

б) Тиазидные.

в) Калийсберегающие.

г) Осмотические.

021. Ингибитором карбоангидразы является:

а) Этакриновая кислота.

б) Ацетазоламид.

в) Спиронолактон.

г) Индапамид.

022. Место действия тиазидных и тиазидоподобных диуретиков:

а) Проксимальные извитые канальцы.

б) Собирательные трубочки.

в) Нисходящий отдел петли Генле.

г) Восходящего отдела петли Генле и дистальные извитые канальцы.

023. Особенностью действия тиазидных и тиазидоподобных диуретиков является:

а) «Низкий терапевтический потолок» - хороший эффект на низких дозах.

б) Дозозависимый эффект – увеличение диуреза при увеличении дозы препарата.

в) Синдром отмены

г) Развитие гипокальциемии.

024. Среди всех тиазидных диуретиков особенностью препарата индапамида является:

а) Лучший профиль безопасности.

б) Более выраженный диуретический эффект.

в) Менее выраженный гипотензивный эффект.

г) Отрицательное влияние на углеводный обмен.

025. Особенностью фармакодинамики ацетазоламида является:

а) Сохранение ионов калия.

б) Выраженное диуретическое действие.

в) Дозозависимый эффект.

г) Снижение внутричерепной гипертензии.

026. При назначении петлевых диуретиков детям с непереносимостью сульфаниламидных производных рекомендуется выбрать:

а) Торасемид.

б) Фуросемид.

в) Этакриновую кислоту.

г) Буметанид.

001. Действие аденозина на проводимость в AV-узле:

а) Кратковременная блокада проведения.

б) Кратковременное улучшение проведения.

в) Не влияют.

002. Среди антиаритмических препаратов I класса фазу реполяризации укорачивают:

003. Препараты Ia класса действуют:

а) Только на предсердия.

б) Только на желудочки

в) На предсердия и желудочки

004. Отличительной особенностью препаратов Ia класса по сравнению с другими препаратами I класса является:

а) Максимально выраженный отрицательный инотропный эффект.

б) Максимальная по продолжительности блокада натриевых каналов.

в) Отсутствие ваголитической активности.

г) Наличие альфа-адреномиметической активности.

005. Препаратами выбора для лечения нарушений ритма, связанных с гиперсимпатикотонией являются:

а) Антагонисты кальция.

б) Мембраностабилизирующие средства.

в) Бета-адреноблокаторы.

г) Местные анестетики.

006. К антиаритмическим препаратам III класса относятся:

а) Блокаторы калиевых каналов

б) Блокаторы натриевых каналов

в) Антагонисты кальция

г) Бета - адреноблокаторы

007. Механизм действия сердечных гликозидов связан:

а) С блокадой натрий-калиевой АТФазы.

б) С активацией ДНК-гиразы.

в) С блокадой фосфолипазы А2.

г) С активацией циклооксигеназы.

008. Фармакодинамические эффекты дигоксина:

а) Положительный инотропный и батмотропный, отрицательный хронотропный и дромотропный.

б) Отрицательный инотропный и батмотропный, положительный хронотропный и дромотропный.

в) Положительный инотропный и хронотропный, отрицательный батмотропный и дромотропный.

г) Отрицательный инотропный и дромотропный, положительный хронотропный и батмотропный.

009. При длительной антиаритмической терапии появление серо-фиолетовой окраски кожи лица и рук, фотодерматита открытых участков кожи, аутоиммунного тиреоидита, интерстициального пневмонита характерно для:

а) Новокаинамида.

б) Соталола.

в) Амиодарона.

г) Пропафенона.

010. Препараты, угнетающие проведение импульса по дополнительным путям:

а) Ia, Ic, III классов.

б) Ib, Ic, II классов.

в) Ia, Ib, Ic классов

г) Ia, III, IV классов.

011. При длительной антиаритмической терапии развитие волчаночноподобного синдрома характерно для:

а) Новокаинамида.

б) Амиодарона.

в) Пропроналола.

г) Пропафенона.

012. Укажите препарат, обладающий фармакодинамическими эффектами всех четырех классов антиаритмиков:

а) Новокаинамид;

б) Лидокаин;

в) Амиодарон;

г) Аймалин

013. Препаратом выбора для лечения желудочковой тахикардии по типу "пируэт" является:

а) Магния сульфат.

б) Амиодарон.

в) Лидокаин.

г) Новокаинамид

014. Препаратом выбора для лечения желудочковой тахикардии является:

а) Лидокаин.

б) Аденозин.

в) Дигоксин.

г) Нифедипин.

015. При назначении комбинированной антиаритмической терапии препараты назначаются:

а) В половинных дозах.

б) В полных дозах.

в) В удвоенных дозах.

016. При врожденном синдроме удлиненного интервала QT с целью профилактики развития аритмий назначают:

а) Препараты класса Ia .

б) Препараты класса Ic.

в) Бета-блокаторы.

г) амиодарон.

017. Пациентам с WPW-синдромом противопоказано назначение:

а) Новокаинамида.

б) Пропафенона.

в) Верапамила.

г) Амиодарона.

018. Препаратом выбора в лечении полной AV-блокады является:

а) атропин.

б) магния сульфат.

в) аденозин.

г) дигоксин.

019. Препарат выбора для лечения дигиталисной интоксикации:

а) Новокаинамид.

б) Лидокаин.

в) Дифенин.

г) Верапамил.

020. При постоянной форме мерцательной аритмии для урежения частоты желудочковых ответов предпочтительно назначение:

а) Амиодарона.

б) Пропафенона.

в) Дигоксина.

г) Верапамила.

001. Механизм действия НПВС связан с блокадой фермента:

а) циклооксигеназа;

б) фосфодиэстераза;

в) 5-липооксигеназа;

г) фосфолипаза А2.

002. Выберите препарат, обладающий наибольшей селективностью в отношении ЦОГ-2:

а) аспирин;

б) ибупрофен;

в) целекоксиб;

г) индометацин;

003. Выберите препарат из группы НПВС не обладающий противовоспалительной активностью:

а) Парацетамол.

б) Ацетилсалициловая кислота.

в) Ибупрофен.

г) Нимесулид.

а) Ибупрофен.

б) Диклофенак.

в) Метамизол.

г) Ацетилсалициловая кислота.

005. Выберите правильный ответ. НПВС:

а) угнетают альтерацию и пролиферацию;

б) угнетают эксудацию и пролиферацию (некоторые препараты);

в) угнетают эксудацию и альтерацию;

г) угнетают все фазы воспаления;

006. Выберите группу препаратов, которая рекомендована к совместному применению с НПВС с целью снижения риска поражения ЖКТ:

а) синтетический простагландин Е1;

б) антациды;

в) прокинетики;

г) гепатопротекторы.

007. НПВС противопоказаны в III триместре беременности так как:

а) Пролонгируют беременность, замедляют родовую деятельность.

б) Вызывают гипотрофию плода.

в) Способствуют нарушению лактации.

008. Пациентам, чьи профессии связаны с повышенным вниманием, не рекомендуется назначать:

а) Индаметацин.

б) Аспирин.

в) Нимесулид.

г) Ибупрофен.

009. В неонатологии для закрытия открытого артериального протока используется:

а) индаметацин.

б) нимесулид.

в) аспирин.

г) фенилбутазон.

010. Синдром Рея возникает при назначении:

а) метамизола.

б) ибупрофена.

в) ацетилсалициловой кислоты.

г) индаметацина.

011. Комбинированное использование НПВС (укажите неверный ответ):

а) возможно для усиления анальгетического и/или антипиретического действия;

б) в ряде случаев ведет к ослаблению эффектов каждого препарата;

в) повышает риск нежелательных реакций;

г) назначается в случае неэффективности монотерапии;

012. Укажите НПВС, с наибольшим влиянием на почечный кровоток и риском нефротоксичности по ишемическому типу:

а) ибупрофен;

б) парацетамол;

в) целекоксиб;

г) диклофенак;

013. НПВС проявляют иммуносупрессивный эффект за счет влияния на:

а) Т-клеточное звено;

б) В-клеточное звено;

в) фагоцитарную активность;

г) капиллярную проницаемость и нарушением контакта антигена с иммунокомпетентными клетками.

014. Наибольшим риском гематотоксичности обладают НПВС из группы:

а) салицилаты;

б) производные индолуксусной кислоты;

в) пиразолидины;

г) производные пропионовой кислоты.

015. При остром отравлении ацетилсалициловой кислотой пациенту назначают:

а) Раствор бикарбоната натрия.

б) 5% раствор глюкозы.

в) Реополиглюкин.

г) Переливание плазмы.

016. Наибольшей минералокортикоидной активностью обладает:

а) гидрокортизон;

б) метилпреднизолон;

в) триамцинолона ацетонид;.

г) дексаметазон.

017. Применение системных глюкокортикостероидов приводит к:

а) лейкопении, эритропении, тромбоцитозу;

б) лейкопении, эритроцитозу, тромбоцитозу;

в) лейкоцитозу, эритропении, тромбоцитопении;

г) лейкопении, эритропении, тромбоцитопении.

018. Влияние глюкокортикоидов на обмен кальция:

а) улучшают всасывание кальция в кишечнике;

б) вызывают гипокальциемию и гиперкальциурию;

в) способствуют накоплению кальция в костях;

г) снижают почечную экскрецию кальция.

019. Системные глюкокортикостероиды при проведении фармакодинамической терапии назначаются:

а) 2/3 дозы в 8.00 и 1/3 в 12.00

б) 1/3 дозы в 8.00 и 2/3 в 12.00

в) 2/3 дозы в 8.00 и 1/3 в 20.00

г) 3 раза в день в равных дозах

020. Укажите системный глюкокортикостероид длительного действия:

а) преднизон;

б) бетаметазон;

в) беклометазона дипропионат;

г) метилпреднизолон.

021. Выберите правильный ответ. Глюкокортикостероиды:

а) практически не влияют на углеводный обмен;

б) усиливают действие вводимого инсулина;

в) усиливают эффект пероральных гипогликемических средств;

г) являются контринсулярными гормонами.

022. Обязательным условием назначения альтернирующей терапии системными глюкокортикостероидами является:

а) отсутствие эффекта от ежедневного назначения глюкокортикостероидов;

б) неэффективность пульс-терапии;

в) стабилизация общего состояния больного;

г) декомпенсированное состояние больного.

023. При проведении заместительной терапии более предпочтителен:

а) преднизолон;

б) метилпреднизолон;

в) дексаметазон;

г) гидрокортизон

024. При лечении бактериального менингита, отека мозга применяется:

а) преднизолон;

б) метилпреднизолон;

в) дексаметазон;

г) гидрокортизон.

025. При 3-х недельном курсе терапии системными стероидами препарат отменяют:

а) Постепенно, по 2.5-5 мг по преднизолону каждые 3-5 дней.

б) Резкой отменой.

в) Постепенно, по 2.5 – 5 мг по преднизолону каждые 2 дня

г) Постепенно, по 10 мг по преднизолону каждые 10 дней.

026. Системные глюкокортикостероиды противопоказаны при:

а) Эпилепсии.

б) Менингите.

г) Тромбоцитопенической пурпуре.

001. Базисная терапия первой линии легкого персистирующего течения бронхиальной астмы включает:

а) Низкие дозы ингаляционных глюкортикостероидов.

б) Кромоны.

в) Антилейкотриеновые препараты

г) Теофиллины длительного действия

002. При обострении бронхиальной астмы препаратом первого выбора на любом этапе оказания медицинской помощи является:

а) эуфиллин.

б) преднизолон.

в) адреналин.

г) сальбутамол.

003. При обострении бронхиальной астмы предпочтительно назначение препаратов:

а) Через небулайзер.

б) Через спейсор.

в) Перорально.

г) Внутривенно.

004. Выберите ингаляционный глюкокортикостероид с наименьшей биодоступностью:

а) флютиказона пропионат;

б) флунизолид;

в) будесонид;

г) триамцинолона ацетонид.

005. При обострении бронхиальной астмы средней степени тяжести обязательно назначение:

а) аминофилина;

б) системного глюкокортикостероида;

в) кромогликата натрия;

г) внутривенное введение?2 –агонистов короткого действия.

а) Антигистаминные препараты.

б) Ингаляционные глюкокортикостероиды.

в) Симпатомиметики.

г) Антихолинергические средства.

007. Снижение объема противовоспалительной терапии возможно при достижении и поддержании контроля бронхиальной астмы в течении:

а) 1 месяца.

б) 3 месяцев.

в) 6 месяцев.

008. Выраженный местный эффект ингаляционных глюкокортикостероидов определяется:

а) высокой липофильностью препаратов;

б) ингаляционным введением препаратов;

в) коротким периодом полувыведения;

г) отсутствием минералокортикоидной активности.

009. Системные глюкокортикостероиды назначаются пациенту с бронхиальной астмой:

а) в качестве альтернативной терапии среднетяжелого течения;

б) как усиление терапии тяжелого течения при неэффективности высоких доз ингаляционных глюкокортикостероидов в комбинации с бета2 –агонистами длительного действия, антилейкотриеновыми препаратами, теофиллинами длительного действия;

в) как средства для купирования приступов одышки;

г) в монотерапии тяжелого течения.

010. Риск развития системных побочных эффектов при применении ингаляционных гдюкокортикостероидов возрастает при назначении доз:

а) выше 2000 мкг/сут;

б) выше 500 мкг/сут;

в) выше 800 мкг/сут;

г) выше 1000 мкг/сут.

011. К селективным бета2 –агонистам длительного действия не относится:

а) формотерол;

б) фенотерол;

в) сальметерол;

г) кленбутирол.

012. Выберите селективный ингаляционный М-холинолитик, применяющийся в лечении бронхообструктивного синдрома:

а) тербуталин;

б) тиотропиум бромид;

в) орципреналин;

г) азаметония бромид.

013. Выберите фармакологическую группу, наиболее предпочтительную для лечения аспириновой астмы:

a) кромоны;

б) системные глюкокортикостероиды;

в) антилейкотриеновые препараты;

г) ингаляционный М-холинолитики.

014. Оптимальный бронходилатирующий эффект теофиллина развивается при поддержании его сывороточной концентрации в диапазоне:

а) 5-10 мкг/дл;

б) 10-15 мкг/дл;

в) 20-35 мкг/дл;

г) 35-60 мкг/дл.

015. Клинический эффект при назначении ингаляционных кортикостероидов больным с астмой обычно отмечается через:

а) 1-2 часа;

в) 4-6 недель;

г) 4-6 месяцев.

016. Применение какого препарата определяет наименьший риск побочных эффектов со стороны сердечно-сосудистой системы:

а) фенотерол;

б) тербуталин;

в) сальбутамол.

017. Ингибитором фермента 5-липооксигеназы является препарат:

а) монтелукаст;

б) зилеутон;

в) сальметерол;

г) недокромил натрия.

018. Выберите бета2-агонист длительного действия:

а) кленбутирол

б) карведилол

в) сальбутамол

г) флютиказона пропионат

019. Серетид - это комбинированный препарат, в состав которого входят:

а) фенотерол + кромогликат натрия;

б) ипратропиум бромид + флютиказона пропионат;

в) сальметерол + флютиказона пропионат;

г) ипратропиум бромид + фенотерол.

020. Назначение бета2 –агонистов в лечении бронхиальной астмы противопоказано при:

а) тиреотоксикозе;

б) эпилепсии;

в) беременности;

г) облитерирующем эндоартериите.

021. Бронхолитический эффект метилксантинов связан с ингибированием фермента:

а) фосфодиэстераза

б) фосфолипаза А2

в) липооксигеназа

г) глутаматсинтетаза

022. Для профилактики развития постнагрузочного бронхоспазма используются все перечисленные препараты за исключением:

а) недокромил натрия;

б) сальметерол;

в) монтелукаст;

г) будесонид.

023. Высокий риск побочных эффектов при применении метилксантинов связан с:

а) узким терапевтическим интервалом

б) невозможностью быстрого выведения препарата форсированным диурезом

в) многочисленными факторами, влияющими на метаболизм

г) верно А и B

д) верно А и C

024. К наиболее значимым фармакодинамическим эффектам ингаляционных кортикостероидов относится все, кроме:

а) торможение синтеза и высвобождения медиаторов аллергического воспаления;

б) потенциирование эффектов эндогенных катехоламинов;

в) восстановление чувствительности адренорецепторов;

г) прямое бронходилатирующее действие.

025. Определите тип фармакодинамического взаимодействия ингаляционных глюкокортикостероидов и бета2 –агонистов длительного действия в лечении бронхиальной астмы:

а) синергизм

б) антагонизм

в) аддитивный эффект

г) не взаимодействуют

026. Пациенту с мерцательной аритмией и бронхиальной астмой для купирования приступов затрудненного дыхания рекомендуется:

а) антилейкотриеновые препараты;

б) ингаляционные М-холинолитики;

в) В2 –агонисты короткого действия;

г) метилксантины.

001. Антитромбоцитарные средства:

б) Тормозят биологическую активность основных факторов свёртывания крови.

в) Блокируют синтез факторов свёртывания крови в печени.

002. Антиагреганты - модуляторы системы аденилатциклаза/цАМФ:

а) Дипиридамол.

б) Тиклопидин, клопидогрель.

в) Ацетилсалициловая кислота.

г) Абциксимаб, эптифибатид, тирофибан.

003. Показанием к назначению аспирина как антиагреганта является:

а) Профилактика тромбозов у больных с ревматическим митральным стенозом.

б) Профилактика тромбозов у больных с язвенной болезнью желудка.

в) Профилактика тромбозов у больных циррозом печени.

г) Профилактика тромбозов у беременных.

004. Какой тип ЦОГ необходимо заблокировать для развития антиагрегантного эффекта?

005. Антиагреганты – блокаторы гликопротеиновых рецепторов для фибриногена GP Iib/IIIa:

а) Абциксимаб, эптифибатид, тирофибан.

б) Дипиридамол.

в) Тиклопидин, клопидогрель.

г) Ацетилсалициловая кислота.

006. Используемые в настоящее время лекарственные формы препаратов из группы блокаторов гликопротеиновых рецепторов GP IIb/IIIa тромбоцитов:

а) Только внутривенные.

б) Только таблетированные.

в) И внутривенные, и таблетированные.

007. Аспирин блокирует циклооксигеназу тромбоцитов:

а) Обратимо.

б) Необратимо.

008. Нагрузочная антиагрегантная доза аспирина составляет:

а) 75-160 мг/сут.

б) 500-1000 мг/сут.

в) 160-325 мг/сут.

009. Противопоказанием к назначению аспирина как антиагреганта является:

а) Профилактика тромбозов у больных с клиническими проявлениями атеросклероза.

б) Язвенная болезнь желудка и двенадцатиперстной кишки.

в) Первичная профилактика ИБС у больных из группы высокого риска.

г) Застойная сердечная недостаточность.

010. К антитромбин-III независимым ингибиторам тромбина относятся:

а) Гирудин.

б) Нефракционированный гепарин,

в) низкомолекулярные гепарины,

г) сулодексид.

011. Механизм действия непрямых антикоагулянтов:

а) Блокируют фермент витамин К-редуктазу.

б) Образуют комплекс с антитромбином III.

в) Блокируют фермент ЦОГ-1.

г) Блокируют фермент витамин К-синтетазу.

012. Нефракционированный гепарин инактивирует следующие факторы свёртывания:

а) IIa, IXa, Xa, XIa, XIIa.

б) IIa, IXa, Xa.

013. Антикоагулянтный эффект нефракционированный гепарин реализует:

а) Непосредственно действуя на факторы свёртывания.

б) Образуя комплекс с тканевым активирующим фактором.

в) Образуя комплекс с антитромбином III.

г) Образуя комплекс с протромбином.

014. Антикоагулянты прямого действия:

а) Тормозят адгезию и агрегацию тромбоцитов.

б) Блокируют синтез факторов свёртывания крови в печени.

г) Растворяют фибриновый тромб.

015. Антикоагулянты непрямого действия:

а) Блокируют синтез факторов свёртывания крови в печени.

в) Тормозят биологическую активность основных факторов свёртывания крови.

г) Растворяют фибриновый тромб.

016. Наиболее чувствительны к инактивации комплексом гепарин/антитромбин III факторы:

017. В результате действия непрямых антикоагулянтов нарушается печёночный синтез следующих факторов свёртывания:

а) II, VII, IX, X.

б) II, VIII, IX, XI.

в) VII, IX, XI, XII.

г) II, VII, XI, XII.

018. К антикоагулянтам прямого действия относятся:

а) Нефракционированный гепарин, низкомолекулярные гепарины, сулодексид.

б) Ингибиторы циклооксигеназы; тиенопиридины; модуляторы системы аденилатциклаза/цАМФ; блокаторы гликопротеиновых рецепторов для фибриногена GP IIb/IIIa.

в) Монокумарины, дикумарины, индандиноны.

г) Активаторы эндогенного плазминогена.

019. Противопоказанием к назначению антикоагулянтов непрямого действия с целью профилактики и лечения тромбоэмболических осложнений является:

а) Язвенная болезнь желудка и двенадцатиперстной кишки в стадии обострения.

б) Мерцательная аритмия.

в) Протезированные клапаны сердца.

г) Тромб в полости левого желудочка.

020. Эффективным методом борьбы с развившимся кровотечением на фоне терапии непрямыми антикоагулянтами является:

а) Свежезамороженная плазма.

б) Викасол.

в) Протамина сульфат.

г) Аминокапроновая кислота.

021. Контролируемый параметр терапии низкомолекулярными гепаринами:

а) Активированное частичное тромбопластиновое время (АЧТВ).

б) Время свёртывания крови.

в) Международное нормализационное отношение (МНО).

022. Контролируемый параметр терапии антикоагулянтами непрямого действия:

а) Международное нормализационное отношение (МНО).

б) Активированное частичное тромбопластиновое время (АЧТВ).

в) Время свёртывания крови.

г) Протромбиновый индекс (ПТИ).

023. Противопоказанием к назначению низкомолекулярного гепарина является:

а) Диабетическая геморрагическая ретинопатия.

б) Подготовка к проведению кардиоверсии у пациентов с мерцательной аритмией.

в) Профилактика и лечение тромбозов глубоких вен нижних конечностей.

г) Профилактика и лечение тромбозов у больных с протезированными клапанами сердца.

024. Фибринолитические средства:

а) Растворяют фибриновый тромб.

б) Тормозят адгезию и агрегацию тромбоцитов.

в) Тормозят биологическую активность основных факторов свёртывания крови.

г) Блокируют синтез факторов свёртывания крови в печени.

025. Фибринолитики противопоказаны в случае:

а) Тромбоза глубоких вен нижних конечностей

б) Острого коронарного синдрома с подъёмом сегмента ST.

г) Тромбоза центральной артерии и вены сетчатки.

001. Выберите наиболее рациональную тактику назначения антацидов:

а) чередование приема всасывающихся и невсасывающихся;

б) использование только всасывающихся антацидов;

в) использование только невсасывающихся антацидов;

г) использование невсасывающихся антацидов при неэффективности всасывающихся антацидов.

002. Наиболее оптимален прием антацидов:

а) перед едой;

б) во время еды;

в) между приемами пищи;

г) вне зависимости от приема пищи.

003. При длительном приеме высоких доз невсасывающихся антацидов возможно появление нежелательных реакций (исключите неверный ответ):

а) повышение риска нефролитиаза;

б) гипергастринемия;

в) гипофосфатемия;

г) деменция при нарушении функции почек.

004. Комбинация гидроокиси магния и гидроокиси алюминия обеспечивает:

а) усиление антацидного действия;

б) усиление послабляющего эффекта;

в) быстрое начало антацидного действия в сочетании с его достаточной продолжительностью;

г) повышение всасывания компонентов препарата.

005. Основной механизм действия ингибиторов протонной помпы:

а) адсорбция соляной кислоты;

б) нарушение образования соляной кислоты;

в) блокада рецепторов, регулирующих секрецию соляной кислоты;

г) нейтрализация соляной кислоты химическим путем.

006. Протонная помпа – это фермент:

а) Н+К+-АТФаза

б) гуанилатциклаза

в) Na+К+-АТФаза

г) фосфодиэстераза

007. Ингибиторы протонной помпы:

а) увеличивают продукцию гастрина;

б) тормозят продукцию гастрина;

в) в низких дозах тормозят, в высоких – увеличивают;

г) не влияют на продукцию гастрина.

008. Выберите препарат из группы ингибиторов протонного насоса:

а) мизопростол

б) пирензепин

в) фамотидин

г) рабепразол

009. Наиболее активный ингибитор цитохрома Р-450:

а) циметидин

б) ранитидин

в) фамотидин

г) роксaтидин

010. Укажите нежелательный эффект, развивающийся на фоне приема фамотидина:

а) гематотоксичность

б) антиандрогенное действие

в) гепатотоксичность

г) синдром рикошета

011. Какое влияние на эндокринную систему оказывают Н2-гистаминоблокаторы 1 поколения?

а) антиандрогенное действие

б) антиэстрогенный эффект

в) антигонадотропный эффект

г) торможение выработки вазопрессина

012. Выберите препарат из группы селективных М1,2-холинолитиков:

а) мизопростол

б) платифиллин

в) цезаприд

г) пирензепин

013. Наиболее выраженной антисекреторной активностью обладают:

а) ингибиторы протонной помпы

б) Н2-гистаминоблокаторы

в) селективные М-холинолитики

г) синтетические простагландины

014. Пилорид – это комбинированный препарат, включающий:

а) ранитидина висмута субцитрат

б) фамотидина висмута субцитрат

в) препараты трехкомпонентной схемы эрадикационной терапии в рекомендованных дозах

г) омепразола висмута субцитрат

015. Основное показание к назначению мизопростола:

а) лечение дуодено-гастрального рефлюкса

б) лечение язвенной болезни желудка

в) совместное применение с НПВС для снижения гастротоксичности

г) лечение функциональной диспепсии

016. Основными фармакодинамическими эффектами мизопростола являются (исключить неверный ответ):

а) уменьшение синтеза соляной кислоты

б) повышение регенерации

в) увеличение выработки простагландинов

г) бактерицидное действие на H. pylori

017. В трехкомпонентной схеме эрадикационной терапии H.pylori у детей старше 14 лет рекомендуется назначить:

а) ингибитор протонной помпы

б) Н2-гистаминоблокатор

в) селективный М-холинолитик

г) синтетический простагландин Е2

а) Амоксициллин + Кларитромицин + Коллоидный субцитрат висмута;

б) Амоксициллин + Кларитромицин + Омепразол;

в) Амоксициллин + Рокситромицин + Метронидазол;

г) Амоксициллин +Кларитромицин + Ранитидин.

019. Абсолютным показанем к проведению эрадикационной терапии является:

а) гипертрофический гастрит;

в) дуоденогастральный рефлюкс;

а) 5 - 7 дней

б) 7 - 10 дней

в) 10 - 14 дней

021. Контроль эрадикации проводится через:

а) 10 дней после терапии

б) 2 недели после терапии

в) 4 недели после терапии

г) 6-8 недель после терапии

022. Укажите препарат не активный в отношении H. pylori:

а) метронидазол.

б) эритромицин.

в) кларитромицин.

г) тетрациклин.

023. Укажите показание к бактериологическому исследованию биоптата слизистой желудка и определения чувствительности H. Pylori к антибиотикам:

а) язвенная болезнь желудка и 12-перстной кишки;

б) рак желудка;

в) длительная терапия НПВС или ГКС;

024. Укажите верное утверждение:

а) домперидон повышает напряжение нижнего пищеводного сфинктера, сократимость антральной части желудка и повышает тонус пилорического сфинктера;

б) домперидон плохо проникает через ГЭБ, что не снижает его терапевтический эффект, но увеличивает количество побочных эффектов;

в) домперидон чаще по сравнению с метоклопрамидом вызывает экстрапирамидные нарушения;

г) домперидон снижает напряжение нижнего пищеводного сфинктера, сократимость антральной части желудка и снижает тонус пилорического сфинктера.

РАЗДЕЛ VIII. КЛИНИЧЕСКАЯ ФАРМАКОЛОГИЯ АНТИМИКРОБНЫХ ПРЕПАРАТОВ И ИХ ПРИМЕНЕНИЕ В ПЕДИАТРИИ.

001. Спектр активности аминогликозидов включает преимущественно:

а) Грамотрицательную аэробную микрофлору.

б) Грамположительную аэробную микрофлору

в) Грамположительную анаэробную флору.

г) Атипичных внутриклеточных возбудителей.

002. Укажите препарат из группы цефалоспоринов эффективный в лечении менингита, вызванного менингококком:

а) цефтриаксон.

б) цефазолин.

в) цефуроксим.

г) цефалексин.

003. Наиболее высокая биодоступность при пероральном приеме характерна для:

а) Амоксициллина.

б) Оксациллина.

в) Ампициллина.

г) Феноксиметилпенициллина.

004. При избыточной массе тела (более 20%) суточная доза аминогликозидов:

а) рассчитывается на фактический вес.

б) рассчитывается на идеальный вес.

в) аминогликозиды противопоказаны к назначению.

005. Выберите препарат активный в отношении энтерококковой инфекции:

а) Амоксициллин.

б) Цефазолин.

в) Азитромицин.

г) Цефипим.

006. Выберите препарат для лечения микоплазменной инфекции у ребенка 6 лет:

а) Доксициклин.

б) Моксифлоксацин.

в) Кларитромицин.

г) Амикацин.

007. Эффективность антибактериальной терапии оценивается:

а) Через 2-3-е суток после назначения препарата.

б) В первые сутки назначения препарата.

в) Через 5-6 суток после назначения препарата.

008. Укажите макролид, на биодоступность которого значимо влияет присутствие пищи:

а) Эритромицин.

б) Спирамицин.

в) Джозамицин.

г) Кларитромицин.

009. Развитие «серого» синдрома характерно при назначении новорожденным:

а) Хлорамфеникола.

б) Гентамицина.

в) Цефтриаксона.

г) Тетрациклина.

010. Укажите препарат выбора для эмпирической терапии внебольничной пневмонии у детей с 1 по 6 месяц жизни, при низкой вероятности атипичных возбудителей:

а) Амоксициллина/клавуланат.

б) Амоксициллин.

в) Гентамицин.

г) Левофлоксацин

д) Бензилпенициллин.

011. Механизм действия макролидов заключается:

а) в специфическом ингибировании синтеза клеточной мембраны

б) в подавлении синтеза белка на уровне рибосом

в) в ингибировании синтеза РНК

г) в нарушении метаболизма фолиевой кислоты

012. Применение клавулановой кислоты в сочетании с амоксициллином позволяет:

а) расширить спектр действия амоксициллина

б) снизить токсичность амоксициллина

в) сократить частоту приема амоксициллина

г) увеличить проникновение амоксициллина через ГЭБ

013. Из перечисленных антибиотиков укажите препарат выбора для лечения инфекции, вызванной метициллинрезистентным стафилококком:

а) ципрофлоксацин

б) имипенем

в) ванкомицин

г) цефипим

014. Какой из перечисленных антибиотиков обладает наименьшим нефротоксическим действием:

а) гентамицин

б) ванкомицин

в) эритромицин

г) стрептомицин

015. Выберите препарат из группы цефалоспоринов, имеющий форму для перорального приема:

а) цефепим;

б) цефиксим;

в) цефазолин;

г) цефотаксим.

016. Препараты из группы цефалоспоринов III поколения отличаются от цефалоспоринов I поколения:

а) расширением спектра активности в отношении грамотрицательной флоры;

б) большей токсичностью препаратов;

в) расширением спектра активности в отношении грамоположительной флоры;

г) меньшей устойчивостью к продукции микроорганизмами бета-лактамаз.

017. Механизм действия фторхинолонов заключается:

а) в специфическом ингибировании синтеза клеточной мембраны;

б) в подавлении синтеза белка на уровне рибосом;

в) в ингибировании ДНК-гиразы и нарушении репликации ДНК;

г) в нарушении метаболизма фолиевой кислоты.

018. Наиболее рациональным в эмпирической терапии внебольничной пневмонии у детей старше 1 года является комбинация аминопенициллинов с:

а) цефалоспоринами III поколения;

б) ингибиторами бета-лактамаз;

в) макролидами;

г) аминогликозидами.

019. Препараты из группы макролидов не обладают активностью в отношении:

а) стрептококков;

б) возбудителей коклюша и дифтерии;

в) золотистого стафилококка;

г) синегнойной палочки.

020. Наиболее характерным побочным эффектом пенициллинов является:

а) нефротоксичность;

б) аллергические реакции;

в) гепатотоксичность;

г) тератогенность.

021. Комбинация гентамицина с ванкомицином считается:

а) рациональной, вследствие повышения антибактериальной активности обоих препаратов;

б) нерациональной из-за риска высокой гепатотоксичности;

в) нерациональной из-за снижения антибактериальной активности;

г) нерациональной из-за риска высокой нейро- и нефротоксичности.

022. Препараты из группы макролидов оказывают:

а) бактерицидное действие;

б) бактериостатическое действие;

в) преимущественно бактериостатическое действие, но в отношении некоторых штаммов действуют бактерицидно;

г) преимущественно бактерицидное действие, но в отношении некоторых штаммов действуют бактериостатически.

023. К группе цефалоспоринов III поколения с выраженной антисинегнойной активностью относится:

а) цефтазидим

б) цефиксим

в) цефтриаксон

г) цефотаксим.

024. Активностью против атипичных возбудителей (микоплазм, хламидий, легионелл) не обладает:

а) ципрофлоксацин

б) доксициклин

в) ампициллин

г) кларитромицин.

025. Укажите антибактериальный препарат, обладающий наибольшей активностью в отношении анаэробной флоры:

а) гентамицин

б) тетрациклин

в) ампициллин

г) метронидазол.


ЭТАЛПНЫ ОТВЕТОВ

РАЗДЕЛ I. ОБЩИЕ ВОПРОСЫ КЛИНИЧЕСКОЙ ФАРМАКОЛОГИИ.

РАЗДЕЛ II. КЛИНИЧЕСКАЯ ФАРМАКОЛОГИЯ СРЕДСТВ, ВЛИЯЮЩИХ НА СОСУДИСТЫЙ ТОНУС. ЛЕЧЕНИЕ АРТЕРИАЛЬНОЙ ГИПЕРТЕНЗИИ У ДЕТЕЙ.

РАЗДЕЛ III. КЛИНИЧЕСКАЯ ФАРМАКОЛОГИЯ АНТИАРИТМИЧЕСКИХ ПРЕПАРАТОВ И ИХ ПРИМЕНЕНИЕ В ПЕДИАТРИИ.

РАЗДЕЛ IV. КЛИНИЧЕСКАЯ ФАРМАКОЛОГИЯ НПВС И ГКС.

ЗАНЯТИЕ V. КЛИНИЧЕСКАЯ ФАРМАКОЛОГИЯ СРЕДСТВ, ПРИМЕНЯЕМЫХ ПРИ СИНДРОМЕ БРОНХООБСТРУКЦИИ У ДЕТЕЙ.

РАЗДЕЛ VI. СРЕДСТВА, ВЛИЯЮЩИЕ НА ГЕМОСТАЗ И РЕОЛОГИЮ.

РАЗДЕЛ VII. КЛИНИЧЕСКАЯ ФАРМАКОЛОГИЯ СРЕДСТВ, ВЛИЯЮЩИХ НА МОТОРИКУ И СЕКРЕЦИЮ ЖКТ.

(=#)РАЗДЕЛ VIII. КЛИНИЧЕСКАЯ ФАРМАКОЛОГИЯ АНТИМИКРОБНЫХ ПРЕПАРАТОВ И ИХ ПРИМЕНЕНИЕ В ПЕДИАТРИИ.

Здравствуйте, дорогие друзья!

Вот и еще одна тема у нас с вами пройдена. Очень надеюсь, что я помогла вам разобраться с лекарственными .

Хотите проверить себя, все ли поняли? Все ли уложилось «по полочкам»?

В таком случае ниже будет тест, который я для вас приготовила.

Как обычно, постаралась сделать его максимально практичным.

В некоторых вопросах фигурируют препараты, которые мы с вами подробно не разбирали. Это я сделала специально. А кто сказал, что их знать не надо? 🙂

Если затрудняетесь, будет повод заглянуть в инструкцию.

Не расстраивайтесь, если результат окажется не очень...

Мне почему-то кажется, что здесь, на моем блоге, бывают только те трудяги фармбизнеса, которые хотят разобраться во всем досконально. Об этом говорят ваши вопросы в комментариях. Так что не опускайте рук и изучайте все, что я здесь выкладываю. Если нужно, проходите одну и ту же тему еще и еще раз.

Я уже говорила вам, что люблю тесты? Это классная форма проверки знаний, чтобы понять, над чем еще нужно поработать.

Мне по своей основной работе приходится их составлять часто.

Не могу сказать, что сотрудники компании, в которой тружусь, от них в восторге. Они всегда напрягаются, поскольку от результатов тестирования что-нибудь, да зависит.

Но здесь, на блоге, никаких нервотрепок не бывает. Его можно проходить хоть 100 раз, чтобы запомнить и понять, что к чему. Тем более, ограничений по времени никаких.

А значит, есть достаточно времени, чтобы подумать над вопросом и без спешки порассуждать.

Например, вот такой вопрос:

«Посетитель жалуется на сильный зуд, жжение в области ануса, иногда бывает кровь в стуле. Какой препарат Вы предложите в первую очередь?»

Ход мыслей будет примерно таким:

«Если сильный зуд и жжение, значит, нужно что-то с гормональным компонентом. Поэтому препараты X и Y отметаем.

Остается два. Если бывает кровь в стуле, значит, препарат с гепарином лучше не давать. Отметаем и препарат Z. Следовательно остается препарат...»

В общем, спокойненько прохОдите тест, видите неважнецкий результат, значит надо что-то еще раз почитать, а потом пройти снова и снова, пока результат вас удовлетворит.

Итак, в тесте 18 вопросов. В каждом возможен только ОДИН правильный вариант ответа. Максимальное количество баллов, которые можно набрать — 18.

Желаю удачи!

Как, друзья, Вам понравился этот тест? А свои результаты? 🙂

Расскажите, что у вас получилось! Было сложно?

Полный сборник тестов в формате «word» с ответами вы можете приобрести .

А я с вами прощаюсь до новой встречи на блоге « ».

С любовью к вам, Марина Кузнецова